Sei sulla pagina 1di 101

TABLE OF CONTENTS

Contenido
1. NÚMEROS ENTEROS ...........................................................................................................................3
2. DIVISIBILIDAD.................................................................................................................................. 11
3. FRACCIONES ................................................................................................................................... 17
4. NÚMEROS DECIMALES ...................................................................................................................... 23
5. PROPORCIONALIDAD ........................................................................................................................ 29
6. LENGUAJE ALGEBRAICO ..................................................................................................................... 39
7. FIGURAS PLANAS ............................................................................................................................. 49
8. SISTEMAS DE ECUACIONES ................................................................................................................. 59
9. FUNCIONES .................................................................................................................................... 69
10. ESTADÍSTICA ............................................................................................................................... 78
11. CUERPOS GEOMÉTRICOS Y VOLÚMENES ............................................................................................ 89
12. PROBABILIDAD ............................................................................................................................ 97

1
1. NÚMEROS ENTEROS
Los números enteros son todos los naturales (0, 1, 2,…) y sus opuestos (-1,-2,-3,…). Su
símbolo matemático es . Pueden representarse en la recta numérica:

-6 -5 -4 -3 -2 -1 0 1 2 3 4 5 6
 Operaciones:
Orden: Usamos los símbolos < y > . Un número es mayor que otro si está más a la derecha
en la recta numérica. Diremos que 3 < 5 y que -3 > -5.

Suma: sumamos positivos por una parte y negativos por otra. Dos números con distinto
signo se restan, y se deja el signo del mayor. Ejemplo: 3 + 2 – 4 – 5 + 3 = 8 – 9 = -1.

Multiplicar: es sumar muchas veces el mismo número. Regla de los signos


Ejemplo: 2+2+2+2 = 2 · 4 = 8. Aquí, los factores son 2 y 4, y su producto 8. +·+=+
Dividir: se usa para repartir en partes iguales. Recuerda que +·-=-
-·+=-
dividendo es igual a divisor por cociente más resto. -·-=+

Potencia: es multiplicar muchas veces el mismo número. Ejemplo: 2 · 2 · 2 · 2 · 2 = 25 = 32.


Aquí, la base es 2 y el exponente 5.

Raíz cuadrada: hay que encontrar un número que al cuadrado sea el radicando (el número
del que queremos sacar la raíz). No hay raíces cuadradas de números negativos.
Por ejemplo, , porque 32 = 9, pero no existe.

Valor absoluto: es el número sin el signo. |-2| = 2, y |+2| = 2.

Cuadrado perfecto: llamamos así a los números naturales al cuadrado. Por ejemplo, 22=4,
32=9, 42=16, 52=25, 62=36, 72=49, 82=64, 92=81, etc.
Ejercicio: Calcula y memoriza los siguientes cuadrados perfectos.

112 = 122 = 132 = 142 = 152 = 252 =

 Orden de las operaciones (jerarquía de las operaciones):

No siempre se hacen las cuentas de izquierda a derecha. Tenemos que hacerlo en este orden:

1) Paréntesis.

2) Potencias y raíces.

3) Multiplicaciones y divisiones

4) Sumas y restas.

3
 Propiedades de la suma

Conmutativa: Podemos sumar en cualquier orden. Por ejemplo: 3 + 5 = 5 + 3 = 8.

Asociativa: al sumar varios números, podemos agruparlos como queramos.


Ejemplo: 3+5+2 = (3+5)+2 = 3+(5+2)=10.

Elemento neutro: Al sumar 0 a un número, se obtiene el mismo número.

 Propiedades de la multiplicación

Conmutativa: Podemos multiplicar en cualquier orden; “el orden de los factores no altera el
producto”. Por ejemplo: 3 · 5 = 5 · 3 = 15.

Asociativa: al multiplicar varios números, podemos agruparlos como queramos. Por


ejemplo: 3 · 5 · 2 = (3 · 5) · 2 = 3 · (5 · 2) = 30.

Elemento neutro: Al multiplicar un número por 1, se obtiene el mismo número.

Distributiva: Al multiplicar un número por una suma, podemos multiplicar primero cada
sumando. Por ejemplo: 2 · (3 + 4) = 2 · 3 + 2 · 4 = 6 + 8 = 14.

 Propiedades de las potencias

Producto de potencias:

- Misma base: se deja la base y se suman los exponentes. Ejemplo: 32·34=32+4=36.


- Mismos exponentes: se multiplican las bases y se deja el exponente. Ejemplo: 23·53=103.

Cociente de potencias: (es casi lo mismo)

- Misma base: se deja la base y se restan los exponentes. Ejemplo: 36·34=36-4=32.


- Mismos exponentes: se dividen las bases y se deja el exponente. Ejemplo: 103 : 53 = 23.

Potencia de potencia: se multiplican los exponentes. Ejemplo: (102)3=106 y ((102)3)5=1030.

Base negativa (entre paréntesis): si el exponente es impar dejamos el signo “-“ y si es par se
quita. Por ejemplo: (-3)2=32=9 y (-2)3 = - 23= - 8.
Mucho cuidado con - 32 = - 9. ¡Si la base NO está entre paréntesis, no se quita el signo!

Exponente 0: el resultado es 1. Ejemplo: 230=1, 30=1.

Exponente 1: se deja solo la base. Ejemplo: 231=23 y 31=3.

4
INTEGER NUMBERS
The Integers are formed by the whole numbers (0, 1, 2,…) and their opposites. Their
mathematical symbol is . We may depict the integers on the number line:

-6 -5 -4 -3 -2 -1 0 1 2 3 4 5 6
 Operations:
Order: We will use symbols < and > . A bigger number will always be found to the right
of a smaller number. We shall say that 3 < 5 and 10 > 2 .

Sum: We add possitives on the one side and negatives on the other. Integers with different
signs: substract them and keep the sign of the largest. Example: 3 + 2 – 4 – 5 + 3 = 8 – 9 = -1.

Multiplication: it is the the repeated addition of the same number. Rules of signs
Example: 2+2+2+2 = 2 · 4 = 8. Here, factors are 2 and 4, and their product 8. +·+=+
+·-=-
Division: it is used any time that distributing in equal parts is needed.
-·+=-
Remember that:
-·-=+
dividend equals divisor times quotient plus remainder.

Exponentiation: is the result of the repeated multiplication of the same number. For
instance: 2 · 2 · 2 · 2 · 2 = 25 = 32. Here, the base is 2 and the exponent 5. (it can be read as 2
raised to the 5th power, or 2 to the 5, or 2 raised to the exponent [of] 5)

Square root: it is to find a number such that squared equals to the radicand (the number
we want to take the root). There is no square root for negative numbers.
For instance, , since 32 = 9, but does not exist.

Absolute value: it is the number without its sign. |-2| = 2, and |+2| = 2.

Perfect square: Any natural number that is the square of another one. For instance, 22=4,
32=9, 42=16, 52=25, 62=36, 72=49, 82=64, 92=81, etc.

Exercise: calculate and memorize the following perfect squares.


112 = 122 = 132 = 142 = 152 = 252 =

 Order of operations (precedence rules):


Accounts are not always performed from the left side to the right. We must follow this order:

1) Parentheses (brackets).
2) Exponents and roots.
3) Multiplication and divisions
4) Addition and subtraction.

5
 Addition properties

Commutative: The order in which two numbers are added does not matter. For instance:
3 + 5 = 5 + 3 = 8.

Associative: when adding several numbers, the order in which additions are performed does
not matter. Example: 3+5+2=(3+5)+2=3+(5+2)=10

Identity element: when adding zero to any number, the quantity does not change.

 Properties of multiplication

Commutative: one can multiply in any order; “the order of the factors doesn’t change the
product”. For instance: 3 · 5 = 5 · 3 = 15.

Associative: When three or more numbers are multiplied, the product is the same regardless
of the grouping of the factors. For example: 3 · 5 · 2 = (3 · 5) · 2 = 3 · (5 · 2) = 30

Identity element: anything multiplied by one is itself.

Distributive: when multiplying a number by an addition, one can multiply first each addend.
For instance: 2 · (3 + 4) = 2 · 3 + 2 · 4 = 6 + 8 = 14.

 Exponent Properties

Product of powers:

- Same bases: add the exponents and keep the common base. Example: 32·34=32+4=36.
- Same exponents: multiply the bases and keep the exponent. Example: 23·53=103.

Quotient of powers:

- Same bases: subtract the exponents and keep the common base. Example: 36·34=36-4=32.
- Same exponents: divide the bases and keep the exponent. Example: 103:53=23.

Power to a power:: multiply the exponents. Example: (102)3=106 and ((102)3)5=1030.

Negative base (in parentheses): if the exponent is odd, then we leave “-“ sign, but if the
exponent is even, we remove it. For example: (-3)2=32=9 y (-2)3 = - 23= - 8.
Watch out! - 32 = - 9. If the base is NOT in parentheses, do not remove the sign!

Zero Exponent: any number raised to the zero power is equal to 1. Example: 230=1, 30=1.

First Power: just keep the base. Example: 231=23, 31=3.

6
OPERATIONS INVOLVING INTEGER NUMBERS - PRACTICE

1. Work out:

a) = b) =

c) = d) =

e) = f) =

g) = h) =

i) - - - - - - - = j)

k) = l) - - - - - - - =

2. Work out:

a) -1 + 8 – 3 · [5 - 2·(3 - 6)] + (-2)= b) =

c) = d)

e) f) – =

g)

h)

i)

j)

Solutions:
1. [a] -10 [b] -14 [c] 5 [d] 0 [e] 6 [f] 26
[g] 5 [h] 10 [i] -8 [j] 8 [k] 2 [l] 15.

2. [a] -28 [b] 17 [c] 118 [d] -5 [e] 8 [f] 27


[g] -5 [h] -11 [i] 18 [j] -22.

7
Integer Numbers - Word Problems

1. From 100 feet above sea level, Juan took off (despegar) in his helicopter and ascended 700
feet. What integer represents Juan's elevation now? [Sol: +800 (above the sea level)]

2. In a trivia competition, Bianca had accumulated 600 points. Then she lost 500 points in the
TV category. What was her score at that point? [Sol: 100 points]

3. While hiking (hacer senderismo), Luis went down 52 meters. If Luis started at 491 meters
above sea level, what is his elevation now? [Sol: 439 m]

4. The Punic Wars began in 264 B.C. and ended in 146 B.C. How long did the Punic Wars last?
[Sol: 118 years]

5. Roman Civilization began in 509 B.C. and ended in 476 A.D. How long did Roman Civilization
last? [It lasted 985 years]

6. Ana carefully tracks her money. Her records indicate she spent $470 on an iron bench and
deposited $430 she made from an online auction (subasta). What was the change in how
much money Ana had? [“-40”; she has $40 less]

7. Marta ended round one of a game show with -460 points. In round two, she lost 757 points.
What was her final score? [Sol: -1,217points]

8. At 11:00 AM, the temperature started dropping 3°F per hour until it reached 59°F at 4:00
PM. What was the temperature at 11:00 AM? [Sol: 74oF]

9. At 7:00 AM, the temperature is 0 degrees Fahrenheit. If the temperature drops 5 degrees
Fahrenheit per hour, what will the temperature be at 10:00 AM? [Sol: -15ºF]

10. A semi-truck weighs 9,250 pounds when empty. After being


loaded, it weighs 21,375 pounds. What is the weight of the
shipment? [Sol: 12,125pounds]

11. A bomb displaces the oil in a well (pozo) from 975 m deep and raises it to a deposit place
48 m above ground level. How far does the oil displace? [Sol: 1,023m]

8
12. In a well (pozo) there is 800 litres of water. At the top there is a pipe (tubería) that deposits
25 l per minute into the well, and at the bottom there is another pipe which withdraws
(retira) 30 l per minute from the same well.
a) Use an integer number to represent what happens each minute with the amount of
water in the well.
b) How many litres of water will there be in the well after 15 minutes of the pipes
functioning?

13. In the Sahara Desert one day it was 136°F. In the Gobi Desert a temperature of -50°F was
recorded. What is the difference between these two temperatures?

14. The following information appears in the label of a bottle of conditioner for the washing
machine: - Its capacity is 18dl (that is to say, 180cl)
- Each dose is a top (tapón) that has a capacity of 5cl.
a) How many doses are there in each bottle?
b) If I use 2 tops for each laundry (colada),
how many laundries can I do with a bottle? And with four bottles?
c) If I do the laundry twice a week, how many weeks does a bottle last?

15. The air temperature in the atmosphere decreases at the rate of 9 ºC every 300 meters. If
today the temperature is 26ºC, what height would a plain have to fly to experience a
temperature of −73 ºC?
Hint: Find the difference of temperatures first.

16. I had €300 in the bank on Monday. On Tuesday I deposited €400. On Wednesday I bought a
€850 TV. On Thursday I paid €500 to the mechanic. I withdraw (sacar dinero) €1200 today. If
I earn €40 each day,
Use an integer number to represent the situation of my account.
Do I have money or do I owe money? How much?

9
2nd ESO Mathematics Exam (trial exam) IES Extremadura

Bilingual section

Surname ______________________________________________ Mark

Name __________________________________

INTEGER NUMBERS
1. (1 pt.) Calculate, using the distributive property:
a) -3·(4 - 1+2)=
b) (6 – 9)·(-5)=
2. (2 pts.) Use the properties of exponents
a) 212·312·512= b) ((-15)2)3:154=
c) (-4)5·35:(-2)5= d) (-2)3·(-2)5:26·30=

3. (4 pts.). Calculate, considering the order of operations.

a) 3 - 2· [3 – 2 ·(- 5) + 4 · ( -3 - 1)] - (-7)=

b) 
20 + 144 : 3  2 81· 2  16 :  4 = 
c) 3  5²  4 : 4  3² +  3
· + 2 =
12 11

d) 3   24 + 52 +16 : 2  30 + 91  23 : 2 2 =
4. (1 pt.) Approximate the following square roots, (writing how many digits they have, and the first one).

a)

b)

c)

d)

5. (1 pt.) Calculate the following square root and its remainder. Check the result: .

6. (1 pt.) 1752 Pines are planted in a nursery (vivero) for reforesting. They sell them in groups of 12
pines, at 4€ each group. How much money do they get?
Answer:

10
2. DIVISIBILIDAD
Múltiplos (de un número): son los que se obtienen al multiplicar el número por algún otro.
Ejemplo: 6 es múltiplo de 3 porque 6 = 3 · 2. Podemos escribir 6 .
También, diremos que 3 es divisor de 6, o que 6 es divisible por 3 (la división es exacta).

Número primo: si sólo es divisible entre 1 y entre sí mismo. Ejemplo: 5 es primo porque
sólo se puede dividir entre 1 y 5.
Criterio de la raíz: para ver si un número es primo, basta probar si es divisible entre alguno
de los primos menores que su raíz cuadrada. Si es menor que 100, basta probar con 2,3,5,7.
Número compuesto: el que tiene varios divisores. Ejemplo: 6 es compuesto porque 3 es
divisor suyo; esto es, porque es múltiplo de 3. El 1 no es primo ni compuesto.

Factorizar un número: es escribirlo como producto de números primos.

 MCD y MCM (de varios números)


El máximo común divisor (MCD) es el MAYOR de sus divisores (comunes).
Si tenemos la descomposición en números primos, se calcula multiplicando los factores
comunes elevados a los menores exponentes. Si no los hay, el MCD es 1, y son primos entre sí.

El mínimo común múltiplo (MCM) es el MENOR de sus divisores (comunes).


Si tenemos la descomposición en números primos, se calcula multiplicando todos los
factores (comunes o no comunes) elevados a los mayores exponentes.

 Criterios de divisibilidad.
Al comprobar la divisibilidad hay que ver si la división es exacta. A veces podemos cambiar
primero el número por otro más sencillo. Los criterios dicen por qué número cambiarlo.
Divisible por Criterio
2 La última cifra (debe ser par: 0, 2, 4, 6, 8).
3 La suma de las cifras.
Nos quedamos con las dos últimas cifras. De esas dos, si la primera es par,
4
podemos quitarla también, y si no, cambiarla por un 1.
5 La última cifra (0 ó 5).
6 Que sea y .
7 Multiplicamos por 2 la última cifra y se resta con lo que queda del número.
9 La suma de las cifras.
11 Sumamos las cifras en lugar par y luego las de lugar impar, y restamos los
resultados.
Para el criterio del 10, 100, etc., basta mirar si acaba en 0, en 00, etc.

11
DIVISIBILITY

 Multiple (of a number): is the product of any quantity and the number. Example: 6 is
multiple of 3 because 6 = 3 · 2. We may write 6 .
We also say that 3 is a divisor of 6, or 6 is divisible by 3 (the division leaves no remainder).

Prime number: is a natural number that has exactly two distinct natural number divisors: 1
and itself. Example: 5 is prime 5 is prime as only 1 and 5 divide it.

Root test: to check if a number is prime, it is enough to check if it is divisible by any prime
number less than its square root. If it is less than 100, just try with 2, 3, 5, 7.
Composite number: a number having several divisors. Example: 6 is composite since 3 is its
divisor; that is to say, since it is multiple of 3. Number 1 is neither prime nor composite.

Factoring a number is the decomposition into a product of powers of prime numbers.

 GCD y LCM (of several numbers)


The greatest common divisor (GCD) is the LARGEST of their (common) divisors.
It can be found using prime factorizations; multiplying all primes common to them raised to
the smallest exponents. If there are none, GCD is 1, and numbers are coprimes.

The least common multiple (LCM) is the SMALLEST number multiple of all them.
It can be found using prime factorizations; multiplying all primes (common or not) raised to
the largest exponents.

 Divisibility rules.
To check divisibility one must verify that the division is exact. Sometimes we can replace it
first for a simpler number. Divisibility rules tell us which number.

Divisor Divisibility condition


2 The last digit is even (0, 2, 4, 6, or 8).
3 Sum the digits.
Examine the last two digits. If the first of them is even, we can remove it. If
4
it is odd, replace it by 1.
5 The last digit is 0 or 5.
6 It is divisible by 2 and by 3
7 Subtract 2 times the last digit from the rest.
9 Sum the digits.
11 Form the alternating sum of the digits.

Rules for 10, 100, etc., just check if it ends in a 0, a 00, etc.

12
PRIME NUMBERS - PRACTICE

1. Complete the following chart using the divisibility conditions:

Number 2 3 4 5 6 7 9 11
252 Yes Yes No Yes Yes No
385 No
4851 No No Yes
2420 Yes Yes No
162 No No
13860 Yes Yes Yes Yes Yes Yes

2. Use the root test to say if 83 is a prime number

3. Use the root test to say if 137 is a prime number

4. Say if 221 is a prime number.

5. Use Erathostenes’ Sieve (Criba deEratóstenes) to find all the prime numbers less than 120.
1 2 3 4 5 6 7 8 9 10 11 12 13 14 15
16 17 18 19 20 21 22 23 24 25 26 27 28 29 30
31 32 33 34 35 36 37 38 39 40 41 42 43 44 45
46 47 48 49 50 51 52 53 54 55 56 57 58 59 60
61 62 63 64 65 66 67 68 69 70 71 72 73 74 75
76 77 78 79 80 81 82 83 84 85 86 87 88 89 90
91 92 93 94 95 96 97 98 99 100 101 102 103 104 105
106 107 108 109 110 111 112 113 114 115 116 117 118 119 120

Solutions

1) Number 2 3 4 5 6 7 9 11
252 Yes Yes Yes No Yes Yes Yes No
385 No No No Yes No Yes No Yes
4851 No Yes No No No Yes Yes Yes
2420 Yes No Yes Yes No No No Yes
162 Yes Yes No No Yes No Yes No
13860 Yes Yes Yes Yes Yes Yes Yes Yes
2
2) It is prime since it is less than 10 , and it is neither divisible by 2, 3, 5 not 7.
3) It is prime since it is less than 122, and it is neither divisible by 2, 3, 5, 7 nor 11.
4) It is not prime, since it is divisible by 13; 221:13=17.
5) 2, 3, 5, 7, 11, 13, 17, 19, 23, 29, 31, 37, 41, 43, 47, 53, 59, 61, 67, 71, 73, 79, 83, 89, 97, 101, 103, 107, 109, 113.

13
6. Use the remainder of the division to answer the following questions:
a) Is 820 multiple of 20? Why?
b) Find the multiple of 20 previous to 820 and the next one.
c) Find all the multiples of 20 between 800 and 900.

7. Use the remainder of the division to answer the following questions:


a) Is 354 multiple of 14? Why?
b) Find the multiple of 14 previous to 354 and the next one.
c) Find all the multiples of 14 between 350 and 450.

8. Use the remainder of the division to answer the following questions:


a) Is 17 divisor of 400? Why?
b) Find the multiple of 17 previous to 400 and the next one.
c) Find all the multiples of 17 between 400 and 500.

9. Find the Least Common Multiple (LCM) and the Greatest Common Divisor (GCD)
a) 20, 35. b) 20, 30 c) 40, 21 d) 12, 24 e) 12, 19 f) 72, 84
g) 90, 120 h) 24, 50 i) 63, 48 j) 42, 60 k) 36, 45 l) 60, 588
m) 46, 98 n) 105, 135 o) 270, 234 p) 315, 420 q) 48, 52 r) 12, 20
s) 24, 18 t) 45, 144 u) 75, 36 v) 63, 27 w) 14, 56 x) 33, 110.

10. Find the Least Common Multiple (LCM) and the Greatest Common Divisor (GCD)
a) 180, 252, 594 b) 924, 1000, 1250 c) 140, 325, 490
d) 725, 980, 1400 e) 2420, 441, 350 f) 952, 1771, 350.

Solutions:
6. a) Yes, since the remainder is 0. b) 820-20=800, and 820+20=840. c) 800, 820, 840, 860, 880, 900.
7. a) No, since the remainder is 4. b) 354-4=350, and 350+14=364. c) 350, 364, 378, 392, 406, 420, 434, 448.
8. a) No, since the remainder is 9. b) 400-9=391, and 391+17=408. c) 408, 425, 442, 459, 476, 493.
3 3 3
9. a) 22·5·7; 5 b) 22·3·5; 2·5 c) 2 ·3·5·7; 1 d) 2 ·3; 22·3 e) 22·3·19; 1 f) 2 ·32·7; 22·3
3 2 3 2 4 2 2 4 2 2 2 2 2
g) 2 ·3 ·5; 2·3·5 h) 2 ·3·5 ; 2 i) 2 ·3 ·7; 3 j) 2 ·3·5·7; 2·3 k) 2 ·3 ·5; 3 l) 2 ·3·5·7 ; 2 ·3
3 3
m) 2·72·23; 2 n) 3 ·5·7; 3·5 o) 2·3 ·5·13; 2·32 p) 22·32·5·7; 3·5·7 q) 24·3·12; 22 r) 22·3·5; 22
3 3 3
s) 2 ·32; 2·3 t) 24·32·5; 32 u) 22·32·52; 3 v) 3 ·7; 32 w) 2 ·7; 2·7 x) 2·3·5·11; 11
3 3
10. a) 22·3 ·5·7·11; 2·32 b) 2 ·3·54·7·11; 2 c) 22·52·72·13; 5
3 2 2 2 2 2 2 2 3 2
d) 2 ·5 ·7 ·29; 5 e) 2 ·3 ·5 ·7 ·11 ; 1 f) 2 ·5 ·7·11·17·23; 7.

14
DIVISIBILITY WORD PROBLEMS
(*) Set these problems up using either the LCM or the GCD.

1. Jeffrey has 36 packets of sugar and 48 packets of artificial sweetener (edulcorante). He wants to divide
them into identical groups, with no packets left over (sin que sobren), so that the groups can be
distributed to some tables at the restaurant where he works. What is the greatest number of groups
Jeffrey can make? How many packets of sugar and how many of sweetener will be on each group?

2. Ana has two pieces of rope (cuerda), one 294 dm long and the other 140 dm long. For a project, she
needs to cut them up to produce many pieces of rope that are all of the same length, with no rope left
over. What is the greatest length, in dm, that she can make them? How many pieces will she have?

3. Veronica has 80 commemorative plates and 144 commemorative spoons (cuchara). She wants to
display them in groups throughout (a lo largo de) her house, each with the same combination of plates
and spoons, with none left over. What is the greatest number of groups Veronica can display? How
many plates and how many spoons will be in each group? [Sol: 16 groups. 5 plates and 9 spoons]
4. Madeline is buying nuts and bolts (tuercas y tornillos/pernos) at a local hardware store (ferretería). The
store sells nuts in packs of 70 and bolts in packs of 84. If Madeline wishes to buy the same number of
nuts and bolts, what is the smallest number of nuts that she can buy? [Sol: 420 nuts]
How many packs of each type will she buy? [Sol: 6 of nuts; 5 of bolts]

5. Lydia is posting 18 flyers (folletos) for the science club and 12 for the music
club. She wants to make all the locations identical, with the same
combination of science club flyers and music club flyers. In addition, she
wants to make sure that no flyers are left over. What is the greatest
number of locations that Lydia can post at? [6 locations]
How many flyers of each type will be on each location? [3 science; 2 music]

6. By coincidence, Pipo's Obedience School trained the same number of dogs and cats last week. The
school teaches dogs in groups of 20 and cats in groups of 12. What is the smallest number of cats the
school could have had? How many groups of each? [Sol: 60. 3 groups of dogs and 5 of cats]

7. A florist has 132 tulips (tulipanes) and 180 carnations (claveles). If the florist wants to create identical
bouquets (ramos) without any leftover flowers, what is the greatest number of bouquets the florist can
make? How many tulips and how many carnations will be in each bouquet? [Sol: 12. 11tul; 15carn.]

8. Antonio and Nadia are writers (escritores). A literary critic notices that the two writers have written the
same number of book chapters during their careers, even though all of Antonio’s books have 18
chapters and all of Nadia’s books have 14 chapters. What is the smallest number of chapters each must
have written? And how many books? [Sol: 126; Antonio: 7 books, Nadia: 9 books]

9. Juan realizes that the number on the classroom door is divisible by 21, and his classmate (compañero de
clase) points out (observa) that it is also divisible by 14. What is the smallest possible number that could
be on the classroom door? Hint: if a number is divisible by 21, then the number is a multiple of 21.

15
3. FRACCIONES
Usaremos las fracciones para escribir divisiones donde no necesitemos calcular el cociente.
Ejemplo: . El conjunto de todas las fracciones se llama números racionales. Su símbolo es .

- Fracción propia: si el numerador es menor que el denominador. Ej.: .


- Fracción impropia: si el numerador es mayor que el denominador. Ej.: .
- Cualquier número es una fracción. Basta poner “1” como denominador. Ej.: 3= .

Propiedad fundamental: dos fracciones son equivalentes (el mismo número) si al multiplicar
en cruz se obtiene lo mismo.

 Operaciones con fracciones:


·2
Amplificación: multiplicar numerador y denominador por el mismo número. .
Reducir a común denominador es amplificarlas para que el denominador sea el MCM.
:3
Simplificación: dividir numerador y denominador por el mismo número. . Cuando
una fracción no puede simplificarse más, es irreducible.

Orden: reducimos a común denominador y comparamos los numeradores.


(*) Si tienen el mismo numerador, es mayor la de menor denominador.

Suma y resta: reducimos a común denominador, y se suma o restan los numeradores.

Multiplicación: se multiplica en línea. .


(*) Para calcular una parte de un número, se multiplica. Ej. de 20€ =

División: se multiplica en cruz. .

Potencia y raíz: se calcula la potencia o raíz del numerador y denominador.

Ejemplo. y .

Después de hacer una cuenta, siempre hay que SIMPLIFICAR el resultado.

17
FRACTIONS
We use fractions for writing a division where we do not need to calculate the quotient.
Example: , which is read as three over four, three fourths or three quarters. The set of all
fractions is called rational numbers. It is represented by the symbol .

(*) A line usually separates the numerator and denominator. If the line is slanting (¾) it is called a
solidus or forward slash. If the line is horizontal, it is called a vinculum or, informally, a “fraction bar”.

- Proper fraction: if the numerator is less than the denominator. Ex.: .


- Improper fraction: if numerator is greater than denominator. Ex.: .
- Any number is a fraction. Just place “1” as denominator. Ex.: 3= .

Fundamental property: we can test if two fractions are equivalents (have the same value) by
cross-multiplying their numerators and denominators (results must be equals). This is also
called taking the cross-product.

 Operations with fractions:


·2
Convert: multiply numerator and denominator by the same number. .
Reduce to common denominator: convert them to turn their denominator into the LCM.
:3
Reduce: divide numerator and denominator by the same number. . When a fraction
cannot be reduced any more, it is said to be irreducible, or in lowest terms.

Order: reduce to common denominator. The larger fraction is the one with the larger
numerator.
(*) If two fractions have the same numerator, then the fraction with the smaller denominator
is the larger fraction.

Addition and subtraction: reduce to common denominator, and add or subtract


numerators.

Multiplication: You multiply across the top and bottom. .

(*) To calculate a fraction of a number, multiply them. Ex. of 20€ =

Division: cross-multiply. .

Power and root: just calculate the power or root of numerator and denominator.

Example: and .

After a calculation, you need to SIMPLIFY, whenever possible.

18
OPERATIONS INVOLVING FRACTIONS – PRACTICE

1. Simplify before multiply:

a) = b) = c) : =

b) = c) = d) =

e) f) g)

2. Calculate. Remember to simplify, whenever it is possible.

a) = b) =

c) = d) =

e) = f) (-5) =

3. Work out

a) = b) =

c) = d) =

e) = f) =

4. Work out:

a) = b) c)

d) e) = f)

Solutions

1. [a] 1 [b] [c] 2. [d] -4 [e] [f] [g] 25 [h] [i]

2. [a] [b] [c] [d] [e] [f] .

3. [a] [b] [c] [d] [e] [f] 18.

4. [a] 2 [b] [c] [d] [e] [f] .

19
FRACTIONS WORD PROBLEMS

1. My class has 24 boys and girls in total. Three fourths of then eat lunch in the canteen
(cantina-cafetería). How many students eat in the canteen?

2. A school buys 60 DVDs for its library. Two thirds of the DVDs are in English. How many
English DVDs were purchased?

3. A kilo of sole (lenguado) costs €12. How much does 3/4 of a kilo of sole cost?

4. A grocer buys a lorry full of oranges and sells half within (en menos de...) the first week and
one third during the second week. What fraction of the oranges has he sold after two
weeks?

5. A zoo has 4 lions and 6 lionesses. What fraction of the total are males? And females?

6. Daniel buys two thirds of a block of cheese and out of this (y de lo que había comprado) he
gives his mother half the block of cheese. What fraction of the whole
block (de todo el queso) of cheese does he have left?

7. Juana buys three quarters of a kilo of pastries (pasteles/bollos) and eats


half of them at lunch. What is the weight of the pastries she has left?

8. The Perez family bought a flat (piso) five years ago. Their initial down payment (entrada)
was one fifth of the flat‘s total value. Over the past five years they have paid off an
additional third of the cost of the flat. What fraction of the cost do they have left to pay?

9. A hiker (excursionista) covers a quarter of his route in the morning and an eighth of it in the
afternoon. What fraction of the route does he still need to cover (to complete the route)?

10. At Luisa’s birthday party, one in three guests is Spanish, one in four is from Portugal and
the rest are from other countries. What fraction of the guests is neither from Spain nor
from Portugal? [Sol: 5/12]

11. Vicente moves forward 3/4 of a metre with each step. How many steps does he need to
take to cover 30 metres? (Hint: distribute those 30m in steps of 3/4 of a metre) [Sol: 40 steps]

12. How many 1/10 litre bottles of perfume can you fill with three half-litre bottles?
(Hint: distribute those 3/2 litres in bottles of 1/10 litres) [Sol: 15 bottles]

13. María bought a TV and is paying for it in eight instalments (plazos). She has already paid
five instalments. What fraction of the total price does she have left to pay? If the TV cost
€872, how much has she paid? [Sol: 3/8. She has paid €545]

14. A bus is carrying 30 adults and 20 children. What fraction are children? [Sol: 2/3]

20
15. 450 students go to a school. 45 of them are in the chess (ajedrez) club. What fraction of the
school’s total students are in the chess club? [Sol: 1/10]

16. In a cycling race, a cyclist has covered 60 kilometres of an 80 kilometres route. What
fraction of the route is left to cover? [Sol: 1/4 of the route (he’s covered 3/4)]

17. The total weigh of one hundred identical bags of coffee is 25 kilos. What fraction of a kilo
does each bag hold? [Sol: 1/4 of a kilo]

18. Beef steaks cost €12 per kilo. How much would three quarters of a kilo cost? [Sol: €9]

19. Three quarters of a kilo of kidney beans (habichuelas rojas) costs €3.60.
How much does one kilo cost? [Sol: €4.80] €3.60

20. One and a quarter kilos of fish cost €10. How much does fish cost per kilo?
(Hint: write as an improper fraction and then distribute €10 among it) [Sol: €8]

21. Three quarters of a kilo of cherries cost €3. How much does one kilo cost? [Sol: €4]

22. A travelling salesman bought 800 kilos of melons and has sold three quarters of them.
How many kilos does he have left? [Sol: 200kg]

23. Tomatoes cost €2 per kilo. What is the price of one and a quarter kilos of tomatoes? [€2.5]

24. The Martinez family bought a dishwasher and is paying for it in instalments (plazos). The
dishwasher cost €460 and the family has paid off three fifths of its total cost so far (hasta
ahora). How much is it left to pay? [Sol: €184. (They’ve paid €276 so far)]

25. Paco spends half his savings (ahorros) on a guitar and two fifths on an amplifier. What
fraction of his savings does he spend in total? [Sol: 9/10]

26. My mother bought a cake yesterday. This morning 3/4 of the cake was left, but tonight only
one eighth is left. What fraction of the cake was eaten over the course of the day? [Sol: 5/8]

27. In a small town, half of the population is over 40 years old and one sixth is over 60. What
fraction of the total population is between 40 and 60 years old? [Sol: 1/3]

28. At an international summer camp, 1/3 of the campers are French, 1/4 are Moroccan and
the rest are Spanish. What fraction of the campers are Spanish? [Sol: 5/12]

29. A bottle of juice can hold 3/4 of a litre. How many litres can 100 bottles hold? [Sol: 75litres]

30. How many kilos of lentils (lentejas) do you need to fill 200 bags that hold 3/4 of a kilo each?
[Sol: 150 kilos]

21
2nd ESO Mathematics Exam (trial exam) IES Extremadura
Bilingual section
Surname ______________________________________________ Mark
Name __________________________________

DIVISIBILITY (4.5 pts)

1. (1.5 pts.) Compute the LCM and GCD of the following numbers:
a) 48, 27 and 42 b) 175 and 605.

2. (1 pt.) Complete the following table using the divisibility conditions:

Number 2 3 4 5 7 9 10 11
294 Yes No No No
1400 Yes Yes No
495 No

3. (1 pt.) Is 137 a prime number? (Justify why using a sentence)


Answer:

4. (1 pt.) Ana is buying nuts (tuercas) and bolts (cerrojos) at a local hardware store (ferretería). The
store sells nuts in packs of 12 and bolts in packs of 10.
If Ana wishes to buy the same number of nuts and bolts, what is the smallest number of nuts that
she can buy? (Remember to write the set up, solving and solution)
Answer (solution):

FRACTIONS (5.5 pts.)


5. (3.5 pts.) Calculate (remember to simplify whenever possible):
3 4 49  1 4 
a) 3    = b) 1   :   2  =
6 8 25  4 3 
2
4 2 1  1 3 5 2 4
c)     4   = d)    ·  
3 5 2  3 2 6 5 2
2 1 5 7
6. (1 pt.) Order the following fractions from least to greatest: , , and
3 2 4 6
7. (1 pt.) In the last maths exam, one third of the class failed the exam and three fifths passed it. The
rest of the class did not attend (presentarse a un sitio) the exam.
a) What fraction of the class did not attend the exam?
b) If there are 30 pupils, how many of them did not attend the exam?

22
4. NÚMEROS DECIMALES
Usaremos unidades decimales (décima, centésima, milésima,…). El conjunto de todos los
números con decimales se llama números reales. Su símbolo es .
La parte entera va antes de la “coma”, y la parte decimal detrás.

 Tipos de decimales:
Exacto: vienen de una división exacta. Sólo tienen algunos decimales.
Periódico: vienen de una división no exacta. Un grupo de decimales (periodo) se repite
infinitamente. Son puros si toda la parte decimal es periódica. Si no, son mixtos.
Irracionales ( ): no son fracciones, y tienen infinitos decimales no periódicos.

 Operaciones (cómo se hacen):


Orden: primero comparamos la parte entera. Después la decimal, rellenando con ceros.
Aproximación: - Truncar: quitamos las cifras decimales que no necesitemos.
- Redondear: truncamos, pero hay que aumentar en uno la última cifra si la
siguiente cifra era mayor o igual que 5.
Suma y resta: hay que colocar las “comas” encima de las comas.
Multiplicación: en el resultado hay que poner tantos decimales como tuvieran los factores.
División: corremos las comas hasta que el divisor no tenga decimales. Después, cuando
bajamos el primer decimal, se pone la “coma” en el cociente.

SISTEMA SEXAGESIMAL
Se usa para medir ángulos (grados, minutos y segundos) y tiempo (horas, minutos y segundos).
Forma compleja: cuando se usan varias unidades para expresar la medida.

h 60 Ejemplos: Un ángulo puede medir 30º 20’ 15’’, y una película durar 1h 20m 10s.
:60 M 60
:60 S Los múltiplos y submúltiplos van de 60 en 60.

23
DECIMAL NUMBERS
We will use decimal units (tenth, hundredth, thousandth,…). The set of all decimal numbers is
called real numbers. It is represented by the symbol .
The integer (integral) part is the part to the left of the point, and the decimal part to the right.

 Types of decimal numbers:


Terminating: they come from a terminating decimal fraction. Decimals stop after a few digits.
Periodic (recurring decimal): they come from a recurring decimal fraction. A group of
decimals (period) repeat forever. We call those which start their recurring cycle
immediately after the decimal point purely recurring. Those that have some extra
digits before their cycles are also called mixed recurring (or eventually recurring).
Irrational ( ): any number which does not stop and does not end with a recurring pattern
(thus they are non-fractional numbers).

 Operations (how to perform them):


Order: compare first the integer part. Then the decimal, filled with trailing zeros.
Approximation: - Truncating : remove the decimal digits you do not need.
(to some
- Rounding: truncate, but increase in one the last digit if the next one was
decimal places)
greater or equal to 5.
Adding and subtracting: line up the terms so that all the decimal points are in a vertical line.
Multiplication: the number of decimal places of the result is the sum of the decimal places in
the factors.
Division: move all decimal points to right to make the divisor a whole number. Put decimal
point in the quotient when you use the first decimal of the dividend.

SEXAGESIMAL SYSTEM
It is used to measure angles (degrees, minutes and seconds) and time (hours, minutes and seconds).
Mixed units: When we use multiple units to express the quantity.

h 60 Examples: An angle can measure 30º 20’ 15’’, and a film can last 1h 20m 10s.
:60 M 60
:60 S Multiples and submultiples are measured by sixtieths.

24
DECIMAL NUMBERS WORD PROBLEMS

1. We have 5 metal balls. Each ball weighs 0.7 grams. How much do they weigh in all?

2. Patricia has $425.82 in her checking account (cuenta corriente). How much does she have
in her account after she makes a deposit of $120.75 and a withdrawal of $185.90?

3. An inch equals 2.54cm. If the diagonal of a TV measures 42 inches, how long is it in cm?
If the diagonal of a TV measures 63.5cm, how many inches does it measure?

4. The Sonia’s Clothes store at the mall (centro commercial) has five employees (Devin,
Abigail, Emily, Kevin, and Rebecca).
This week they worked 42, 23.5, 46, 48, and 28 hours. They are paid by the hour. Each
employee is paid at a different hourly rate ($19.25, $16.60, $18.80, $7.45, and $11.75).
Find how much does each employee receive and how much it is in all.

5. A clock gains (se adelanta) 2m 15s every day. How much will gain in a month (30 days)?

6. Juan's basketball practice started at 4:15 P.M. The team practiced offense for 45 minutes
and defense for 1 hour and 15 minutes. What time did Juan's basketball practice end?

7. A land developer is splitting up 769.4 hectares of land to form 2 identical properties. What
size will each property be? [Sol: 384.7ha]

8. Ellen wanted to buy the following items: A DVD player for $49.95, a DVD holder for $19.95
and a personal stereo for $21.95. Does Ellen have enough money to buy all three items if
she has $90 with her? [Sol: No, since they cost $91.85]

9. Expenses for the Rodriguez family vacation were $978.50 for hotels, $489.95 for meals,
$162.79 for transportation, and $76.12 for souvenirs. Estimate the total cost by rounding
to the nearest one (redondea a unidades). [Sol: $1,707.36]

10. Olivia used 0.6 kilograms of almonds to make 5 batches of snack mix. How many kilograms
of almonds (on average) were in each batch? [Sol: 0.12kg on average]

11. A train left Federico's town and travelled for 3 hours to Danville. Then it travelled 1 hour
and 45 minutes and arrived in Stafford at 11:00 A.M. What time did the train leave
Federico's town? [Sol: At 6:15 A.M.]

25
12. A train left Lucy's town at 2:45 P.M. The train travelled for 5 hours and 45 minutes to
Manchester. Then it travelled 1 hour to Somerville. What time did the train arrive in
Somerville? [Sol: It arrived at 9:30P.M.]

13. What is the total cost for 2 kilograms of cashews (anacardos),


1.6 kilograms of pecans, and 0.8 kilogram of walnuts (nueces)?
[Sol: £8.584]

14. How much more does a wheelbarrow (carretilla) cost than a


hammer? [Sol: £30.73]

15. Ana started playing a video game at 11:15 A.M. It took her 30
minutes to beat the first level and 0.4 hours to beat the second
level. Ana played the final level for 1 hour and 45 minutes and
finished the game. What time was it when Ana stopped playing
the game? [Sol: It was 13:54]

16. How much money does Pedro need to buy 9 portable DVD
players, an MP3 player and 7 video game systems? [£1,349.26]

17. On Thursday afternoon at camp, Maggie played basketball


and went swimming before dinner. She started playing basketball at 2:45 P.M. and played
for 2.6 hours. Then she swam for 1 hour and 20 minutes. Dinner lasted for 1 hour. What
time did dinner end? [Sol: At 7:41P.M.]

18. John has £176.50. Does he have enough to buy a


telescope and a case of beakers (vaso de precipitado)?
[Sol: Yes, since they cost £168.71]

19. At soccer practice on Saturday morning, IES Extremadura's


team practiced dribbling for 15 minutes and practiced shooting for 30 minutes. Then, they
played a scrimmage game (partidillo, “pachanga”) for 0.12 hours before practice ended at
11:30 A.M. What time did IES Extremadura’s soccer practice start? [Sol: At 10:37:48 A.M.]

20. If Alicia buys 1.25kg of bran flakes and 5.6kg of wheat flakes,
how much will she spend? [Sol: £10.1275]

26
2nd ESO Mathematics Exam (trial exam) IES Extremadura

Bilingual section

Surname ______________________________________________ Mark

Name __________________________________

Decimal Numbers and Sexagesimal System

1. (1 pt.) Write as a fraction (if possible) and say what type of number they are.

a) 1'23= b) 2'66666....=

c) - 2'131313...= d) 1'23333...=

2. (0,5 pts.) Add and subtract

a) 2.6+0.37+21= b) 5.81-10.6=

3. (0,5 pts.) Round to tenths: a) 4'56= b) 3'213=

4. (1 pt.) Multiply:

a) 7.334 · 1.2= b) 0.081 · 0.42=

5. (1 pt.) Divide using two decimal places 16’016:2’24=

6. (1 pt.) Work out using two decimal places:

7. (2 pts.) Calculate using two decimal places:

a) 4'2 – 3'4 · 5'1 + 2= b) (15 – 6’35): 2’5=

8. (1,5 pts.) Write using mixed units:

a) 6.53h= b) 8121’’=

9. (1,5 pts.) Roberto goes to the market with 62.81€ and buys 2.6kg grapes at 1.80€ each kg. 0.58Kg
bananas at 2.15€ per kg, a hake (merluza) that weighs 850g and costs 11.45€ each Kg, and a chicken of
1.25Kg, that costs 5.95€ per Kg. How much has he spent? How much does he have left?

a) Setup:

b) Solving

c) Solution (answer):

27
5. PROPORCIONALIDAD
- Magnitud: cantidad que se puede medir con números, usando unidades de medida.

- Razón entre dos magnitudes: es su cociente. Ejemplo: .

- Proporción: es la igualdad entre dos razones. El número obtenido se llama constante de


proporcionalidad. Ejemplo: es una proporción y la constante es 2.5. Además, 5 y 10 se
llaman extremos, y 2 y 25 medios.

 Relación entre dos magnitudes:


Directa: si una aumenta, la otra también. Si además son proporcionales, para calcular una se
multiplica la constante por la otra, y al doble de una le corresponde el doble de la otra, etc.
Inversa: si una aumenta, la otra disminuye. Si además son proporcionales, para calcular una
se divide la constante por la otra, y al doble de una le corresponde la mitad de la otra, etc.

 Cálculo del cuarto proporcional:

Para resolver algo como , se usa la propiedad fundamental: ( 5·x=1 · 20 )

“El producto de los medios es igual al producto de los extremos”.

 Resolución de problemas. Se pueden hacer de dos formas:


1ª: calcular la constante de proporcionalidad “reducción a la unidad”.
2ª: hacer el planteamiento usando la regla de tres.

- Para el Reparto Proporcional, la cantidad a repartir se corresponde con el total.

 PORCENTAJES:

Cálculo: se multiplica (el porcentaje en forma de número). Ejemplo: 5% de 20 = 0.05 · 20=1.


Problemas: El total se corresponde con el 100%. Puede usarse la regla de tres, o si no:.
- Calcular los porcentajes y luego hacer las operaciones.
- Hacer las operaciones con los porcentajes y luego calcular los números.

 PROPORCIONALIDAD GEOMÉTRICA:

- Teorema de Tales: cuando dos rectas cortan a varias paralelas, los segmentos que
determinan son proporcionales.
- Figuras semejantes: cuando tienen la misma forma. Tienen sus lados son proporcionales.
Para las áreas, se multiplica por la razón al cuadrado, y para los volúmenes, la razón al cubo.
- En los mapas a escala, se hace una semejanza del dibujo con la realidad.

29
PROPORTIONALITY
- Magnitude: quantity that can be measured with numbers, using measure units.

- Ratio between magnitudes: it is their quotient. Example: .

- Proportion: It is a statement that two ratios are equal. The number obtained is called
constant of proportionality. Example: is a proportion and the constant is 2.5.
Furthermore, 5 and 10 are called extremes, and 2 and 25 means.

 Relationship between magnitudes (types of variation):


Direct: the greater the 1st, the greater the 2nd. If they are also proportional, to calculate one,
multiply the constant by the other. If one doubles, the other will also double, etc.
Inverse: the greater the 1st, the smaller the 2nd. If they are also proportional, to calculate
one, divide the constant by the other. If one doubles, the other will become half as large, etc.

 Finding the fourth proportional:

To solve something like , one uses the fundamental property: ( 5·x=1 · 20 )

“the product of the means is equal to the product of the extremes”.

 Problem solving. There are two ways:


1st: find the constant of variation “unitary method”.
2nd: set the problem up using the rule of three.

- For Proportional Distribution, the amount to share out corresponds to the total.

 PERCENTAGES:

Calculation: multiply (with the percentage as a number). Example: 5% of 20 = 0.05 · 20=1.


Problems: The total always corresponds to 100%. We can either use the rule of three, or:
- Calculate percentages first and then perform the calculations.
- Perform calculations with percentages, and then write the result as a number.

 GEOMETRIC PROPORTIONALITY:

- Thales Theorem (intercept theorem): if two intersecting lines are intercepted by some
parallels, the segments obtained are proportional.
- Similar objects: when they have the same shape. In that case, their sides are proportional.
For areas, we multiply by the ratio squared, and for volumes, the ratio cubed.
- In scaled maps we make a similarity of the drawing with reality.

30
PROPORTIONALITY WORD PROBLEMS

1. A cyclist takes 6 minutes to cover 2 kilometres. How long would he take to cover 15
kilometres? [Sol: 45 minutes]

2. A factory produces 1,400 pairs of trousers in one week (7 days). How many pairs of
trousers does it produce in one month (30 days)? [Sol: 6,000 pairs]

3. 3 workers build a wall in 12 hours. How long would it have taken for 6 equally productive
workers? [Sol: 6 hours]

4. A fountain pumps out (sacar-bombear) 120 litres of water in 20 minutes. How many litres
does it pump out in half an hour? [Sol: 180 litres]

5. It takes 14 hours for a faucet (AmE: grifo) with a flow of 18 liters per minute to fill a
reservoir (embalse) with water. How long will it take if its flow is reduced to 7 liters per
minute? [Sol: 36 hours]

6. Water running from a tap (BrE: grifo) fills a 60 litre jug with water in 18 minutes. How long
will it take to fill an 80 litre jug? [Sol: 24 minutes]

7. A farmer has enough grain to feed 60 cattle (ganado) for 10 days. He sells 10 cattle. For
how many days will the grain last now? [Sol: 12 days]

8. A 300 gram wedge (cuña-trozo) of cheese costs €6. How much would a 350 gram wedge of
cheese cost? [Sol: €7]

9. The members of the Drama Club are going to paint the scenery for their play. They figured
that it would take 9 of them 10 days to paint the scenery. If they need the scenery in 6
days, how many members do they need to help paint? [Sol: 15 members]

10. Seven tickets to the zoo cost €35. How much will it cost to buy 25 tickets? [Sol: €125]

11. You can make 6 litres of juice with 10 kilos of oranges.


How many litres of juice can you make with 75 kilos of oranges? [Sol: 45 litres]
How many kilos do you need to make 120 litres of juice? [Sol: 200kg]

12. In 4 hours a power shovel (excavadora) digs (cava) a hole that is 10


metres deep. How deep is the hole it digs in 9 hours? [Sol: 22.5m]

13. 24 people can construct a house in 15 days. But the owner would like to finish the work in
12 days. How many workers should he employ? [Sol: 30 workers]

31
14. The groceries (comestibles) in a home of 4 members are enough for 30 days. If a guest
comes and stays with them, how many days will the groceries last? [Sol: 24 days]

15. Tomás, who can type 30 words per minute, can type a paper in 2.6 hours. How long will
take Victoria to type that paper if she can type 26 words per minute? [Sol: 3 hours]

16. 35 carts are needed to transport the entire grain in a warehouse (almacén) at the rate of 8
bags a truck. But the truck drivers objected saying it was too heavy and carried only 7 bags
a truck. How many more trucks are needed to transport the grain? [Sol: 5 more carts]

17. For a scientific research study, 56 beavers (castor) were captured, tagged (etiquetado),
and set free. Later, the researchers returned and found 36 tagged beavers out of the 540
beavers they counted. What is the best estimate for the beaver population? [Sol: 840]

18. A group of 2ºESO volunteers helped determine the lizard (lagarto) population of a local
park. First, they marked 70 lizards. Then, counted a total of 400 lizards in the park, 20 of
which were marked. To the nearest whole number, what is the best estimate for the lizard
population? [Sol: 1,400 lizards]
19. A race car does two laps around a circuit in 9 minutes.
How long will it take to do 25 laps? [Sol: 112.5 min]
How many laps will do in three quarters of an hour? [Sol: 10 laps]

20. The legs of a right triangle measure 24 m and 10 m and its hypotenuse is 26 m. What is the
length of the legs of a similar triangle whose hypotenuse is 52 m? [Sol: 48m and 20m]

21. An 8 ft tall telephone booth (cabina) casts (proyecta) a 4 ft long shadow. Find the height of
a lawn ornament that casts a 2ft shadow. (*) Hint: Set it up using a picture. [Sol: It is 4ft long]

22. A 6 ft tall tent standing next to a cardboard box casts


(proyecta) a 10 ft shadow. If the cardboard box casts a
shadow that is 5 ft long then how tall is it? [Sol: It is 3ft tall]

23. Two wheels are connected by a transmitter belt. The first has
a radius of 25 cm and the second, 75 cm. When the first has
reached 300 turns, how many turns will the second have completed? [Sol: 100 turns]

Proportional distribution

24. Distribute €153 between two brothers proportionally to their final marks: Pedro: 8, Emi: 9.
[Sol: Pedro €72, Emi: €81]

25. A company is distributing €840 gratification among three of its workers, proportionally to
the extra hours they have spent. Juan: 6h, Ana: 10h, María: 12h. [Sol: €180, €300, €360]

32
26. A hospital is sending 20 patients to other hospitals. Distribute them proportionally to the
available (disponible) beds. Hospital A: 25 beds. Hospital B: 10 beds, Hospital C: 15 beds.
[Sol: 10 patients, 4 patients, 6 patients]

27. Distribute €153 profit (beneficio) among three shop assistants, proportionally to the hours
spent in the shop. Angela: 4h, Rodrigo: 3h, Lidia: 2h. [Sol: €68, €51, €34]

28. Three individuals contribute $5,000, $7,500 and $9,000 which is together invested into a
stock. When the stock (acciones) has made $6,450 on top of the capital (de beneficios), it is
withdrawn, how much will each individual receive? [Sol: $1500, $2250, $2700 on top]

Percentages

29. 95% of a herd (rebaño-manada) of 340 animals are sheep and the rest are goats. How
many sheep and how many goats are there? [Sol: 323 sheep and 17 goats]

30. In spring a reservoir contained 5,000 m3 of water, but in summer the reservoir contains
80% less water. How many cubic metres of water does it contain in summer? [Sol: 1,000m3]

31. A hotel has 50 rooms and 35 of them are occupied. What percentage is that? [Sol: 70%]

32. In the English exam, Paula has 76 correct answers out of 80. What percentage of the
answers is correct? What is her mark out of 10? [Sol: 95%; 9.5points]

33. A store pays €32 for a briefcase and marks the price up by 5%. What is the new price? [33.6]

34. The cost of an item is £175, which does not include VAT at 20%. After VAT is included,
what is the total cost? [Sol: the total cost is £210]

35. The manager of a store orders antique clocks that cost £330 and sells them for £627. What
is the mark-up (aumento) percentage? [Sol: It is a 190% of the price. So the mark up is 90%]

36. Adela buys a skirt that normally costs €80. At the till (caja) they give her a 10% discount.
How much is the discount in euros? How much does she pay? [€8 discount. She pays €72]

37. A cake weighs 1,200 grams. 10% of the cake’s weight is butter. How many grams of butter
are there in the cake? [Sol: 120 grams]

38. 12 tables in a restaurant are reserved. If the reserved tables represent 75% of the total
number of tables in the restaurant, how many tables does the restaurant have? [Sol: 16]

39. Julián has read 80 pages of a novel. If those 80 pages represent 25% of the total pages in
the book, how many pages does the book have? [Sol: It’s got 320 pages]

40. Paola spent 10% of her savings when she bought a CD for €15. How much did she have
saved before buying the CD? [Sol: Her savings were €150]

33
Scale maps

41. Find the distance between Riverside and Milton if they are 12 cm apart on a map with a
scale of 4 cm : 21 km. [Sol: 63km]

42. A model house is 12 cm wide. If it was built with a scale of


“3 cm:4 m” then how wide is the real house? [Sol: 16m wide]
43. Find the actual distance between Louberg and Luluville (in miles),
considering the map on the right. [Sol: 10 miles]

44. Two towns are 15cm apart on a map. If the scale for this map is 1 cm
equals 160 km, how many miles apart are the two towns? [2400km]

45. If the actual distance of 1500 km is represented by 5 cm on a map, what is the actual
distance represented by 12 cm? [Sol: 3,600km]

46. This is a plan we have drawn up for a flat using a scale of 1:250.
a) Find its actual measurements, and the actual surface of the flat.
(Note: use a ruler to measure this paper)

b) If we are charged €15.50 for tiling (embaldosar) each m2 of the floor,


how much is it for the whole flat?

47. On a map of the California missions, two of the missions are located 12 centimetres apart.
If the map uses a scale of 2 centimetres: 7 kilometres, how far apart are the two missions
in real life? [Sol: 42km]

48. This is the floor plan of a flat. It is drawn to a scale 1:200.


Calculate the dimensions of the entire flat as well as its rooms.

34
GEOMETRIC PROPORTIONALITY – PRACTICE

1. Use similarity of figures to work out the missing side lengths. Round to tenths.
[1] [2] [3] [4]

Sol: 4.25 and 7.2


Sol: 2.3 and 2.5 Sol: 3.9 and 5.5 Sol: 2.7 and 2.7

[5] [6] [7]

6.3

Sol: 3.9 and 4.7 Sol:6.4 and 7.6 Sol:7.8 and 5.8

[8] [9] [10]


8.7
13.4

Sol: 4.3 and 6.5 Sol: 2 and 4.1


Sol: 7.3 and 11.1

[11] [12] [13]

Sol: 4.5 and 2.6 Sol: 2.6 and 6.5 Sol: 3.4, 4.7 and 2.7, 3.8, 4.6

[14] [15] [16] 3

8 8 10
Sol: 5.4, 4.3 and 3.6, 4.3, 3.5 Sol: 7.8, 6 and 7.7 and 3.7
Sol: 4.9, 11 and 7.7 and 5

[17] [18] [19]

Sol: 5.3, 4.5, 5.2


Sol: 7.8 and 3.9, 2.6 Sol: 3 and 7.6, 14.1

35
2. Find the ratio of similarity (r). Then use it to find the surface of the larger base (A) and the
volume of the big pyramid/cone (V). Finally, work out the frustum’s volume (F).
[Calcula la razón de semejanza (r). Luego úsala para encontrar el área de la base mayor (A) y el volumen de la
pirámide/cono grande (V). Por último, deduce el volumen del tronco (F)].

a) b) 4cm c) 3
12cm 1.2cm 2cm
10.7cm3 4cm 1.8cm2

8cm2
8cm 8cm

9.3cm3
2
7cm

d) e) f) 2.5m
3cm 4m 3
10m
2.75cm3 7.5m
3
2
15m 5m
3cm2 6cm 6m
2
6m

g) 9.17dm3 2.5dm h) i)
2cm
5.5dm2 2cm3 3cm3

3cm2 4.5cm2 5cm


6.25dm 2.5cm

5cm

j) k) 9m l) 1cm
3m 3
0.44cm
1.75cm2 3.5cm
1m
7m2 7m3
0.28m2 0.4m
3
0.04m

Solutions:
2 3 3 2 3 3
a) r=2, A=32cm , V=85.6cm , F=74.9cm . b) r=3, A=63cm , V=251.1cm , F=241.8cm .
2 3 3 2 3 3
c) r=4, A=28.8cm , V=76.8cm , F=75.6cm . d) r=2, A=12cm , V=22cm , F=19.25cm .
3 3 3 3
e) r=1.5, A=13.5m2, V=25.31m , F=17.81m . f) r=2, A=60m2, V=80m , F=70m .
2 3 3 2 3 3
g) r=2.5, A=34.38dm , V=143.28dm , F=134.11dm . h) r=2, A=12cm , V=16cm , F=14cm .
3 3 3 3
i) r=2.5, A=18cm2, V=24cm , F=21cm . j) r=2.5, A=1.75m2, V=0.63m , F=0.59m .
2 3 3 2 3 3
k) r=3, A=63m , V=189m , F=182m . l) r=3.5, A=21.44cm , V=18.87cm , F=18.43cm .

36
2nd ESO Mathematics Exam (trial exam) IES Extremadura

Bilingual section

Surname ______________________________________________ Mark

Name __________________________________

PROPORTIONALITY.

1. (0.5 pts.) On a High-School, all groups have 25 pupils. Explain if the number of pupils going
on a trip is proportional to the quantity of groups enrolled on the trip.
Relate both magnitudes using a formula.

2. (1.25 pts) A car takes 6 hours to travel some route at a speed of 90km/h. How long would it
take it to travel that route, driving at a speed of 60 km/h?

3. (1.25 pts) A fabric softener bottle (bote de suavizante) costs 2,16€. We find an offer where
we pay only a 75% for the second bottle. How much do two bottles cost?

4. (1,25 pts) A pair of trousers cost 30€ last week. Now they are offered with an 80% off. What
is the price now?

5. (1.25 pts.) After a 12% discount, we have paid 44€ for a videogame. What was its price
before the discount?

6. (1.25 pts.) On a map, the scale is 1:20.000. Measuring on the map, the distance between
two towns is 12cm. What is the actual distance?

7. (1.25 pts.) Find the length of segment x:

2.5 cm
3 cm

8 cm

37
8. (2 pts.) On a international athletics championship, 35% of the contestants are
Europeans; one fourth are Americans, 15% Africans; one fifth Asiatic; and the rest from
Oceania.
- What percentage are Americans? And Asiatic? And from Oceania?
- We want to distribute the sports hall surface (which is divided into 100 equal parts),
proportionally to the number of contestants per continent. Represent in this chart
the fraction of contestants corresponding to each continent (you may –puedes- use a
different colour or shading for each continent).

9. (1.25 pts.) Three friends have earned 150€ playing sports lottery.. If Juan put 2€, Carlos
5€ and Antonio 3€. How much do each of them receive?
(The distribution will be proportional).

38
6. LENGUAJE ALGEBRAICO
- Expresión algebraica: números y letras “variables” unidos por operaciones matemáticas.
- Valor numérico: resultado de sustituir por números las letras de una expresión algebraica.
- Monomio: producto de un número “coeficiente” por varias letras “parte literal” (se admiten
exponentes en las letras pero no raíces de letras, o letras en el denominador, etc.). El grado
del monomio es la suma de los exponentes.

 Producto: juntamos las letras y sumamos los exponentes de las que son iguales. Ej: x2y4·xy3=x3y7.

 Suma y resta de monomios semejantes (con la misma parte literal): se suman o restan los
coeficientes. Ej.: 2x+5x=7x, y 2x-5x=-3x.

- Polinomio: suma y/o resta de varios monomios “términos”. Si hay dos términos, es un
binomio; si hay tres, trinomio, etc. Se escriben ordenados, del grado mayor al menor.
El monomio de mayor grado da el grado del polinomio. Su coeficiente se llama principal.
Término independiente: un número que aparece sin letras. Su grado es 0.
Ej: 3x2-5x+1: trinomio de segundo grado, coeficiente principal 3 y término independiente 1.

 Operaciones con polinomios: (el polinomio debe ir entre paréntesis)

Signo “+”: se quita el paréntesis y se deja todo como está. Ej.: +(3x2-5x+1)= 3x2-5x+1.

Signo “-“: se cambia el signo de cada término. Ej.: -(3x2-5x+1)= -3x2+5x-1.


Producto por un número: se usa la propiedad distributiva. Ej.: 2·(3x2-5x+1)= 6x2-10x+2.

ECUACIONES
- Ecuación: Igualdad entre expresiones algebraicas que sólo se cumple para algunos valores de
las letras, que se llaman solución. Resolver la ecuación es encontrar las soluciones.
Las letras se llaman incógnitas. El grado es el mayor grado de los términos.
La parte de la izquierda primer miembro, y la de la derecha, segundo miembro.

 Resolución de ecuaciones de primer grado (ecuaciones lineales):

1. Quitar paréntesis.
(reduciendo primero a común denominador, o si es
2. Quitar denominadores. una igualdad de fracciones, se multiplica en cruz).
3. Transponer y agrupar términos (se les cambia el signo al cambiarlos de miembro).
semejantes.
4. Despejar la incógnita. (Si un número multiplica a un miembro, pasa al otro
dividiendo, y viceversa)

39
 Resolución de ecuaciones de segundo grado (ecuación cuadrática):
CASOS POSIBLES:
1. La incógnita siempre aparece con el mismo grado: se hace como con las de primer grado, y
luego se hace una raíz. En la raíz cuadrada hay que poner ±.
Ejemplo: x2=9; . Soluciones x=+3 y x=-3.

2. Producto igual a 0: se resuelve cada factor por separado.


Ejemplo: para (x-1)·(x-2)=0, se resuelve x-1=0 y x-2=0. Las soluciones son x=1 y x=2.

3. Se puede factorizar: si no hay término independiente, se saca la incógnita factor común.


Ejemplo: x2-3x=0. Factorizamos x·(x-3)=0. Las soluciones son x=0 y x=3.

4. Ecuación “completa”:

ax2+bx+c=0. Soluciones:

Se calcula más rápido si hacemos primero el discriminante .

40
ALGEBRAIC LANGUAGE
- Algebraic expression: numbers and letters “variables” joined by mathematical operations.
- Numerical value: the result of replacing letters by numbers in an algebraic expression.
- Monomial: algebraic expression where a number “coefficient” is multiplied by letters “literal
part” (exponents are allowed on letters but not square roots, nor a letter in a denominator,
etc.). The degree of a monomial is the sum of the exponents.

 Product: join the letters and add the exponents of letters alike. Ex: x2y4·xy3=x3y7.

 Addition and subtraction of like monomials (with the same literal part): add or subtract
coefficients. For instance: 2x+5x=7x and 2x-5x=-3x.

- Polynomial: addition and/or subtraction of several monomials “terms”. If there are two
terms, it is a binomial; if there are three, it is a trinomial, and so on. We write them ordered
from the greatest degree to the least.
The monomial with the greatest degree gives us the degree of the whole polynomial. It is
called the leading term and its coefficient is called leading.
Independent term: a number without letters. Its degree is 0.
Ex: 3x2-5x+1: second degree trinomial, principal coefficient 3 and independent term 1.

 Operations involving polynomials: (when the polynomial is in parentheses)

“+” sign: remove the parentheses and leave everything as it is. Ex.: +(3x2-5x+1)= 3x2-5x+1.

“-“ sign: change the sign of the terms. Ex.: -(3x2-5x+1)= -3x2+5x-1.
Product by a number: we use the distributive property. Ex.: 2·(3x2-5x+1)= 6x2-10x+2.

EQUATIONS
- Equation: Equality of two algebraic expressions that is correct for only certain values of the
letters, called solution. To solve an equation is to find its solutions.
Letters are called unknowns. The degree is the greatest of the terms’ degrees.
The left side is the first member, and the right side, the second member.

 Solving first degree equations (linear equations):

1. Eliminate parentheses.
(reduce first to common denominator, or if it is an
2. Eliminate fractions. equality of fractions, cross multiply.).
3. Transpose and combine like terms. (changing the sign when changing to the other side).

4. Get the unknown by itself. (If a number multiplies on one member, goes to the
other dividing it, and vice versa)
41
 Solving second degree equations (quadratic equations):
POSSIBLE CASES:
1. The unknown always appears with the same degree: it is solved as the first degree ones
and then, one takes a root. With square roots one has to write ±.
Example: x2=9; . Solutions: x=+3 and x=-3.

2. A product equals 0: solve each factor separately.


Example: for (x-1)·(x-2)=0, we solve x-1=0 and x-2=0. Solutions are x=1 and x=2.

3. It is possible to factorise: if there is no independent term, pick out common factor the unknown.
Example: x2-3x=0. We factorise as x·(x-3) =0. Solutions are x=0 and x=3.

4. “Complete” equation:

ax2+bx+c=0. Solutions:

It is computed faster if we first work out the discriminant .

42
FIRST DEGREE EQUATIONS. REVISION EXERCISES
1. Solve the following equations. (Hint: use cross-multiply).
Resuelve las siguientes ecuaciones (Pista: multiplica en cruz):

x  5 2x  3 2x  1 3 7  x 13  x
a)  b)  c) 
2 3 4x  2 5 3 6
Sol: x=9 Sol: x=-11/2 Sol: x=1
x  11 x  5 x2 x  1 10
d)  e) 6 f) 
6 3 3 x3 6
Sol: x=1 Sol: x=20 Sol: x=9

2. Solve the following equations. (Hint: reduce to common denominator).


Resuelve las siguientes ecuaciones (Pista: reduce a común denominador):

x  2 x 1 x x 1 x 1 x 1 x  3
a)   1 b)   1 c)   1
9 3 2 3 4 6 4
Sol: x=7 Sol: x=19/7 Sol: x=5

x x x x2 3(1  x) x 1 x 1 x  3
d)   5 e)  10  f)    2
2 3 4 3 2 8 6 5
Sol: x=60/7 Sol: x=-55/7 Sol: x=7

3. Solve:

a) b) x-3·(2x+5·(1-x))=2x-10 c)

d) e) f) 1-(2-(3x+1)-x))=2x-19

g) h) i)

j) k) l)

Solutions:
a. x=4. b. x=1. c. x=0. d. x=-5. e. x=-5. f. x=-5.
g. x=1. h. x=3. i. x=2. J. x=8. k. x=5. l. x=-2.

43
SECOND DEGREE EQUATIONS. REVISION EXERCISES

1. Solve:
a) x2 27=2·(x2 1) b) 3x2 -4x2 4-2x2 =-23 c) 5x2 -60 x2 =4(x2 3)

d) x2 6 3x2 =6 5x2 -1 e) 6x2 -3=-2·(5-3x2 ) 3x2 -20 f) -2x2 3(2x2 -100)=63 x2

g) h) - i) - .

2. Solve
a) (x-4)·(2x-6)=0 b) (5x-3)(x+1)=0 c) (4-x)(x+5)=0
d) (x2-25)(x+2)=0 e) (3x2-12)(5x2-45)=0 f) x(4x-1)=0
g) (7x+14)·x=0 h) x·(2x2-50)=0 i) 2x2-6x=0
j) 5x2-3x=0 k) 4x2-36x=0 l) 5x2=10x.

3. Solve
a) 2x2 -8x-10=0 b) 3x2 -3x-18=0 c) 4x2 -32x 48=0

d) 5x2 -15x 10=0 e) x2 -16x 63=0 f)

g) -3x2 24x-48=0 h) -4x2 4x 8=0 i) -5x2 20x-15=0

j) (3x 2)·(2x 1)=12 k)(x-4)·(3x-4)=-5 l) (2x-1)·(3x-2)=12

m) n) o) x2 =2x 35

p) 2x(x 1)=3(1-x2 ) q) 3x2 (x 1)=5x-x2 r) 3x(x-3)=-2(x 1)

s) (5x-1)(2x-4)=3x(x 1)-8 t) 2 x2 -9)=(x-3)·(4x 2) u) x·(2-x)-20=(5-2x)·(x 2).

Solutions
[1] a. x=±5 b. x=±3 c. x=±6 d. x=±1 e. x=±3 f. x=±11
g. x=± h. x=± i. No solution (there is no square root for negative numbers)

[2] a. x=3, x=4 b. x=3/5, x=-1 c. x=-5, x=4 d. x=±5, x=-2. e. x=±2, x=±3 f. x=0, x=
g. x=-2, x=0 h. x=0, x=±5 i. x=0, x=3 j. x=0, x=3/5 k. x=0, x=9 l. x=0, x=2.

[3] a. x=-1, x=5 b. x=-2, x=3 c. x=2, x=6 d. x=1, x=2 e. x=7, x=9 f. x=-3, x=4
g. x=4 (double) h. x=-1, x=2 i. x=1, x=3 j. x=-2, x=5/6 k. x=3, x=7/3 l. x=2, x=-
m. x=1, x=4 n. x=-3, x=2 o. x=-5, x=7 p. x=-1, x=3/5 q. x= (double) r. x=2, x=
s. x=3, x= t. x=2, x=3. u. x=-6, x=5

44
REVISION AND SUMMARY OF TECHNIQUES

1. Operations involving polynomials:


a) - · - · = b) - - · -

2. Equality of fractions. Cross multiply.

a) b) c) -
- -

3. Equations involving fractions. Remove denominators.


- -
a) - b) - - c)

4. The unknown is always with the same degree. Solve as in first degree equations.
a) - - - b) - - · - c) - -

5. Product equals to cero. Solve each factor separately.


a) - · b) ( - · - =0 c)

6. It is possible to take out common factor.


a) x2 -5x=0 b) 3x2 -9 =0 c) 2x2 2

d) - - - = e) 3x(x 1)-6(x 1)=0 f) 2x(x 5) 10(x 5)=0

7. Second degree equation formula.


a) - · - b) - · - c) - ·

d) - - e) · - f) · -

Solutions:
[1] a) b) [2] a) x=2 b) x=1 c) x=-3
[3] a) x=2 b) x=-1 c) x=-1 [4] a) x=±3 b) x=±2 c) x=±3
[5] a) x=3, x=-2 b) x=±1, x=2 c)x=-3 (remember there is no square root for negative numbers)
[6] a) x=0, x=5 b) x=0, x=3 c) x=0, x=-6 d) x=4, x=2 e) x=2, x=-1 c) x=±5

[7] a) x=3, x=- b) x=3, x= c) x=-1, x=2 d) x=-1, x= e) x=2, x= - f) x=-3, x=5.

45
ALGEBRAIC LANGUAGE AND EQUATIONS- WORD PROBLEMS

1. Determine an expression representing the total cost to employ carpenters at €20 per hour
and electricians at €27 per hour.

2. Sari and her friends are going to play miniature golf. There are p people in the group. Each
person pays €5 for a round of golf and together they spend €9 on snacks. Write an
expression that represents the total amount that Sari and her friends spent.

3. At the Beltway Outlet store, you buy x computer games for $13 each and a magazine for
$4. Write an expression in simplest form that represents the total amount of money you
spend.

4. The bicycle path at the park is a loop (bucle-circular) that covers a distance of m miles.
Jorge biked 2 loops each on Monday and Wednesday and 3 loops on Friday. On Sunday
Jorge biked 10 miles. Write an expression in simplest form that represents the total
distance that Jorge biked this week.

5. Find the area and the perimeter of these polygons. Simplify the answers.

a) b) c)

6. When a factory makes t bicycles in a month, the gross profit (beneficios brutos) on each
bicycle is 25 + 2t dollars.
a) Write an expression for the total gross profit the factory makes in a month that it
produces t bicycles.
b) What is the gross profit if the factory makes 40 bicycles?

7. A number and the same number plus one add up to 43. What is the number?

8. A number and the same number minus 12 add up to 18. What is the number?

9. If you add thirteen to two times a number, you obtain a result of 99. What is the number?

10. There are 31 people in a café. There are 5 more men than women. How many men and
how many women are there?

46
11. In a pride of 13 lions (familia de 13 leones) there are 3 more females than there are males.
How many lions and how many lionesses are there?

12. On a farm, the horns (cuernos) and legs of the cows add up to 30. How many cows are
there on the farm?

13. Juana bought two pens and a marker for a total of €5. What was the price of each item if a
marker costs fifty cents more than a pen?

14. The base of a rectangle is three centimetres longer than its height. Its perimeter is 38 cm.
What are the dimensions of the rectangle? (*) Use a drawing to set the problem up.

15. Calculate the length of the sides of this isosceles triangle, given that its
perimeter is 47 cm. x+4

16. A box of figs weighs one more kilo than a box of strawberries. Together,
three boxes of strawberries and two boxes of figs weigh 12kg. How
much does each box weigh?
x

17. A bowl of ice-cream costs eighty cents more than a pasty (empanadilla). At snack-time,
María and Ana bought one bowl of ice-cream and two pasties for a
total of €4.40. How much does a pasty cost? How much does a bowl
of ice-cream cost?

18. At the market, Meyer buys a bunch of bananas for $0.35 per pound
and a frozen pizza for $4.99. The total for his purchase was $6.04.
How many pounds of bananas did Meyer buy?

19. Laura is making a patio in her backyard using paving stones (losa). She buys 44 paving
stones and a flowerpot (macetero) worth $7 for a total of $73. How much did each paving
stone cost?

20. A taxi service charges you $1.50 plus $0.60 per minute for a trip to the airport. The
distance to the airport is 10 miles, and the total charge is $13.50. How many minutes did
the ride to the airport take?

21. If we add five to the double of a number, we get twenty three. What is the number?

22. We know that there are 171 animals at the zoo. If there are the double of tigers than
panthers, how many are there of each species?

47
2º ESO Mathematics Exam (trial exam) IES Extremadura
Bilingual section

POLYNOMIALS AND ALGEBRAIC LANGUAGE.

1. (2.5 pts.) Compute:

a. (1.25 pts.)  2·(9 x  7 y)  (3 y  12 x) 

b. (1.25 pts.) (2 x  3) ·(4 x 2  6 x  3) 

1 2
2. (2 pts.)  . Check the solution. Solution:
2x 1 x  5

4 x  3 2( x  3) 3x  1
3. (2 pts.)   . Solution:
6 3 2
4. (2 pts.) Solutions:

5. (1.5 pts.) Word Problem: (Setup, Solving and solution using a sentence are mandatory)

At home, I have three times CDs than DVDs. If I own 52 discs in all,
how many discs of each type do I own?

48
7. FIGURAS PLANAS
- Teorema de Pitágoras: en todo triángulo rectángulo, el cuadrado de la a
hipotenusa es igual a la suma de los cuadrados de los catetos. b
c
a 2 = b 2 + c2

Terna Pitagórica: si a, b, c son enteros. Ej: (3,4,5); (5,12,13); (7,24,25); (8,15,17),…

 ÁREAS
h
b
Paralelogramos: base por altura A=b·h.

- Para el rombo y la cometa, A = (D y d son las diagonales). D


d
Triángulo: A = . b
h
Trapecio: A = B

Polígono regular: A = . (apotema: distancia del centro al lado).

- En general, hay que descomponer la figura en otras de áreas conocidas.


apotema

 FIGURAS CIRCULARES

- Longitud de la circunferencia de radio r: L= 2r. Para un arco de a grados: L = · 2r.

- Área del círculo: A=r2. Para un sector circular de a grados: A = r2.

 ÁNGULOS

- Los ángulos de todo triángulo suman 180º, y los de un polígono convexo, 180 · (lados-2).

- En un polígono regular, el ángulo central mide .

- Ángulos en la circunferencia:
Central Inscrito Semiinscrito Interior Exterior Circunscrito

- Así que todo ángulo inscrito en una semicircunferencia es recto.


(Segundo Teorema de Thales)

49
PLANE FIGURES
- Pythagorean Theorem: In a right triangle the square of the hypotenuse a
is equal to the sum of the squares of the legs (the other two sides). b

a 2 = b 2 + c2 c

Pythagorean Triple: when a, b, c are integers. Ex: (3,4,5); (5,12,13); (7,24,25); (8,15,17),…

 AREAS h
b
Parallelograms: base times height A=b·h.

- For rhombus (diamond) and kite, A = (D and d are the diagonals). D


d
Triangle: A = . (Half the product of the base and height) b
h
Trapezium: A = B

Regular Polygon: A = . (apothem: distance from centre to sides).

- Usually, one has to break the figure into others of known areas. apothem

 CIRCULAR FIGURES

- Circumference of a circle of radius r: L= 2r. For an arc of a degrees: L = · 2r.

- Area of a circle: A=r2. For a circular sector of a degrees: A = r2.

 ANGLES

- The sum of angles in a triangle is 180º, and in a convex polygon is: 180 · (sides-2).

- In a regular polygon, the central angle measures .

- Angles in a circle:
Central Inscribed Semi-inscribed Interior Exterior Circumscribed

- Therefore, any angle inscribed in a semicircle is a right angle.


(Second Thales’ theorem)

50
AREA AND PERIMETER. REVISION EXERCISES
1. Find the area and the perimeter
a) b) c) d)

2. Calculate the length of the lines (and the perimeter in d)


a) b) c) d)

Sol: 3.6 units 4.24 units 5.1 units 2·5+2·1.41=12.82 units

3. Find the area and perimeter. [Hint: you may need the Pythagorean Theorem].
a) b) c)

Sol: A=108cm2. P=48cm h=15; A=495in2; P=100in. A=420ft2; P=90ft.

d) e) f) g)

2 2
Sol: A=39.8in ; P=24.85in A=285.12cm ;P=80.94cm

h) i) j)

2 2
A=52.1cm ; P=34.73cm A=56in ; P=36in

51
4. Calculate the area and perimeter of the following figures. [Each square measures 1cm].

a) b) c)

3cm

2 2
Sol: A=29.42cm ; P=40.50cm Sol: A=49cm ; P=36.49cm

d) e)
Sol: A=28.14cm2; P=34.57cm

[Perimeters calculated including inner sides]

Sol: A=23cm2; P=35.26 Sol: A=31.57cm2; P=34.46cm

5. Calculate the area and the perimeter:


a) b) c)

20cm 20cm 10cm

Sol: A=163.28cm2; P=62.8cm Sol: A=188.4cm2; P=62.8cm Sol: A=78.5cm2;P=47.1cm

d) e) f)
8dm

12cm
16cm

Sol: A=150.72cm2; P=49.12cm Sol: A=264.60cm2; P=49.07cm Sol: A=75.36cm2;P=45.68cm.

6. Find the unknown (using an equation) and the perimeter.

a) 2
b) c) 12m
2
d) e) 11.57dm
2

36cm x
x+4 x+2 24cm2 x+3
9.42dm2
x x x
x

Hint: x=6cm x=2dm x=2m x=6cm x=2dm


Sol: P=24cm P=13.42dm P=16m P=24m P=15.14dm

52
AREA AND PERIMETER EXERCISES.
(*) Use 3.14 for number .

Relationship between radius, side, and apothem in regular polygons.

No. sides Knowing the side Knowing the apothem Knowing the radius
5 a=0,69·side r=0,85·side side=1,45·a r=1,24·a a=0,81·r side=1,18·r
6 a=0,87·side r = side side=1,15·a r=1,15·a a=0,87·r side = r
7 a=1,04·side r=1,15·side side=0,96·a r=1,11·a a=0,90·r side=0,87·r
8 a=1,21·side r=1,31·side side=0,83·a r=1,08·a a=0,92·r side=0,77·r
9 a=1,37·side r=1,46·side side=0,73·a r=1,06·a a=0,94·r side=0,68·r
10 a=1,54·side r=1,62·side side=0,65·a r=1,05·a a=0,95·r side=0,62·r

1. Compute area and perimeter, considering the previous table:

a) b)

4m 6cm

5cm
c) d)

2. Compute the area and perimeter of this figure 10cm

60cm 30cm

30cm 80cm

53
3m
3. Work out the area

a) b) 2,5 m
1,6dm
1,5 m

1dm 3,6dm
6,3 m
2 cm
2dm 5dm c)
6 cm

1,4dm

1dm 8 cm 4 cm
d)
2,6dm
3dm

3dm

3dm Sol:
A=174.065dm2
2dm
2dm
2dm

2dm 2dm 3dm


4. Compute the area and the perimeter:

a) b) 4m

10cm Sol:
Sol: A=135.75m2
A=178.5cm
2 10m P=65.39m

c)
2cm 6cm
3.75m
2cm 12m 5m
2cm

d) 10 dm e)
4cm 4cm

3cm
6cm

6 dm

54
PLANE FIGURES - WORD PROBLEMS

1. A cardboard (carton) envelope for a CD is a square with an area of 171.61 square


centimetres. What are the dimensions of the envelope? Round to the nearest tenth.

2. Carmen wants to buy fencing (valla) to enclose a square garden with an area of 500 square
metres. Each metre costs 4€. How much fencing does she need to buy? How much is it?

3. Berta has a piece of paper measuring 8.5 inches by 8.5 inches. If she folds the paper
diagonally in half, how long is the folded side? Round to the nearest tenth. [1 inch=2.54cm]

4. A wire (cable) 10 meters long is supporting a utility pole (palo). The wire is
anchored (anclado) to the ground and is attached to the pole 9 meters above
the ground. What is the distance from the bottom of the pole to the point
where the wire is attached to the ground? Round to the nearest tenth.

5. Rocío wants to build a ramp that will rise 4 feet over a horizontal distance of 20 feet. How
long will the ramp be? Round to the nearest tenth. [1foot=30.48cm]

6. Melody bicycles 6 kilometres west to get from her house to school. After school, she
bicycles 8 kilometres north to her friend Linda's house. How far is Melody's house from
Linda's house, measured in a straight line? [Sol: 10km]

7. A hummingbird's nest (nido de un colibrí) is 16 meters high in a tree and a flower is on the
ground 12 metres away from the base of the tree. How far will the hummingbird need to
fly to get from its nest to the flower? [Sol: 20m]

8. Dave is building a slide for his kids. The ladder is 3 meters tall and the slide is 5 meters
long. What is the distance between the ladder and the bottom of the slide? [Sol: 4m]

9. Stacy's drink coaster (posavasos) has a diameter of 6 cm. What is the coaster's area?

10. Estimate the area of the top of a round pizza that has a diameter of 16 inches. [Sol: 201in2]

11. The lens of a telescope has a surface of 3.14 m2. What is the lens's diameter? [Sol: 2m]

12. A cathedral has a large, circular stained-glass window (vidriera). It has an area of 12.56m2.
What is the window's diameter? [Sol: 4m]

55
13. Norwood is 12 kilometres due north (justo al norte) of the airport, and Morristown is 16
kilometres due east of the airport. How far apart are Norwood and Morristown? [Sol: 20km]

14. Luisa wants to make the International Marine Signal flag shown
which represents the number six. What is the area of the flag?

15. Arkansas has a shape that is similar to a trapezoid with bases of about 182 miles and 267
2
miles and a height of about 254 miles. Estimate the area of the state. [Sol: 157,023miles ]

16. Two parallel streets are cut across by two other parallel streets as
shown in the figure, cutting off a parcel of land in the shape of a
parallelogram. Each ft2 costs 4€ Find the area and the price of the
2
parcel of land. [Sol: 85,000ft , and costs 340,000€]

17. Each side of a square office is 3 meters long. It will cost €49.38 per square meter to
replace the carpet in the office. What would be the total cost to replace the carpet?

18. A school wants to buy a chalkboard which measures 1 meter by 2 meters. The chalkboard
costs $42.00 per square meter. How much will the chalkboard cost? [Sol: It will cost $84]

19. The perimeter of a rectangular shop in the mall (centro commercial) is 84 meters. The
shop is 24 meters long. How wide is it? [Sol: It is 18m wide]

20. Consider Jaco’s preliminary design of his vacation house at the right. Round to the nearest
tenth if necessary.
a) What type of figure is bedroom 1? Find its area.
b) What is the area of the bedroom 2? What figures
did you use to find the area?
c) What is the area of the bathroom?
d) What is the area of the living room?
e) What is the area of the den? What would the
area of the den (studio) be if the semicircular
window were removed and replaced with a flat
window?
f) What is the area of the kitchen? If Jaco adds a
rectangular cooking island in the middle of the
kitchen with dimensions 6 feet by 4 feet, how many
square feet of walking space will be left?

56
2nd ESO Mathematics Exam (trial exam) IES Extremadura

Bilingual section

AREA, PERIMETER AND PYTHAGOREAN THEOREM.

5dm 5dm 5dm 5dm 5dm


1 (4 pts.) Compute the area of the following figures:
3m 5dm
a) b)

2,5 m
30dm 4dm
1,5 m

6dm
5m

2 (1 pt.) A circular carpet has a diameter of 4 metres:


a) Compute its radius, draw the carpet and compute its area.
b) If the square metre of material costs 100€, how much does the carpet cost?

3 (2 pts.) Compute the area and the perimeter of the following circular sector:
36º

3m

4. (1 pt.) A squared skating rink (pista de patinaje) has 90m2 surface. We know that the
smaller side measures 8 metres. How much does the other side measure?
(Setup including a drawing is required)

5. (1 pt.) A rectangular schoolyard (el patio de un colegio) measures 30x50 metres. In the
Physical Education speed test, pupils must run the diagonal of the schoolyard.
What is the distance that pupils have to run in the exam?
(Setup including a drawing is required)

6. (1 pt.) Compute perimeter, height and area for this triangle


(*) Draw its height. 6 cm 6 cm

8 cm

57
8. SISTEMAS DE ECUACIONES
 SISTEMA DE ECUACIONES: es cuando tenemos varias ecuaciones de las que buscamos una
solución común. Si las ecuaciones son de primer grado, el sistema se llama lineal.
- Para dar la solución del sistema, hay que dar un valor para cada incógnita.

 RESOLUCIÓN DE SISTEMAS LINEALES DE DOS ECUACIONES Y DOS INCÓGNITAS.

MÉTODOS ALGEBRAICOS:
Se elimina una incógnita y queda una ecuación de una incógnita. La resolvemos.
Después, se sustituye el valor obtenido en una de las ecuaciones y se resuelve lo que quede.

- Método de SUSTITUCIÓN: despejamos una incógnita en una ecuación y sustituimos en la otra.


- Método de IGUALACIÓN: despejamos la misma incógnita en las dos ecuaciones. Hacemos una
nueva ecuación igualando los resultados.
- Método de REDUCCIÓN: sumamos las ecuaciones de manera que desaparezca una incógnita.
Puede que primero haya que multiplicarlas por algún número.

MÉTODO GEOMÉTRICO:
Cada ecuación lineal se dibuja como una recta del plano (usando una tabla de valores). La
solución es el punto donde se cortan las rectas.

59
EQUATIONS SYSTEMS
 EQUATIONS SYSTEM: it is when we have several equations and we are looking for a common
solution. If the equations are of first degree, the system is called linear.
- In order to find a solution for the system, we have to give a value for each unknown.

 SOLVING LINEAR SYSTEMS OF TWO EQUATIONS AND TWO UNKNOWNS.

ALGEBRAIC METHODS:
Eliminate one unknown in order to get a “one variable equation” that we solve.
Then, we substitute the value obtained on one of the equations and solve the remaining
equation.

- SUBSTITUTION method: isolate one variable in one equation and substitute in the other.
- ALGEBRAIC EQUATION method: isolate the same variable on each equation. Make another
equation equalling the results.
- ELIMINATION (ADDITION) method: add the equations so that one variable disappears. You may
first need to multiply them by some number.

GRAPHING METHOD:
Draw each linear equation as a line on a coordinate plane (using a table of values). The
solution is the point where the lines meet.

60
SYSTEMS OF EQUATIONS. EXERCISES
1. Solve the following systems:

3x  5 y  5  3x  2 y  3  4 x 3  y  1  5 
  
a)  b)  c) 
2 x  y 12  4 x  3 y  2  3  y  1  2 x  7

x y 2x  y  2x  y 
  5   x 1 
3 5 2 5
 
d)  e) 
x y 5x  2x  y 
1   6  3x  5 
2 6  5 

2) Solve using the substitution method (sustitución)


-
- - - -
a) b) c)
- -
Sol: x=6, y=1. Sol: x=-6, y=-2. Sol: x=9, y=1.

- -
- - -
d) e) f)

Sol: x=2, y=-1. Sol: x=-2, y=5. Sol: x=1, y=3.

3) Solve using the elimination method (reducción)

- - - - -
a) b) c)
- -
Sol: x=-5, y=-2. Sol: x=10, y=2. Sol: x=-9, y=-12.

- - - - -3 5 2 - 2 -1
d) e) - f) 2 -1
- 3
4

Sol: x=3, y= 2. Sol: x=3, y=-2. Sol: x=-1, y=-5.

61
4. Solve using both the algebraic equation method and the graphing method:

a) y=2x+3 b) x=y+4 c) 3x+y=11

y=6-x x=2y-1 y-4x=-10

Sol: x=1, y=5 Sol:x=9, y=5 Sol: x=3, y=2

5. Solve and check the solution:

a) x+y=5 b) x+y=6 c) 3x+2y=11

x-y=1 3x-2y=8 5x-3y=-7

Sol: x=3, y=2 Sol:x=4, y=2 Sol: x=1, y=4

Word Problems

6. Find two numbers which sum a total of 9 and a difference of 3. (Set this problem up using an
equation system). [Sol: 6 and 3.]

7. Marta and Sonia are going together to a hardware store. Marta buys 1 kg of wood paint and
2 kg of wall paint with a total price of 70€. Sonia buys 2 kg of wood paint and 2 kg of wall
paint with a total price of 80€.

a) How much is the price of each type of paint? (Express this problem in an equation
system)

b) Meanwhile, Javier buys 3 kg of wood paint and 5 kg of wall paint. How much does Javier
have to pay?

62
EQUATION SYSTEMS – PRACTICE
1. Solve the following equation systems.

a) b)

c) d)

e) f)

g) h)

i) j)

k) l)

2. Solve the following equation systems.

a) b)

c) d)

e) f)

g) h)

i) j)

Solutions:
1. [a] x=2, y=3. [b] x=-3, y=1. [c] x=2, y=5 [d] x=-3, y=10 [e] x=3, y=1 [f] x=5, y=3
[g] x=-2, y=1 [h] x=1, y=4. [i] x=7, y=7 [j] x=1, y=-1 [k] x=5, y=10 [l] x=2, y=-3.

2. [a] x=3, y=2. [b] x=4, y=-1. [c] x=5, y=1 [d] x=4, y=2 [e] x=2, y=-1 [f] x=3, y=6
[g] x=3, y=6 [h] x=2, y=5. [i] x=1, y=-2 [j] x=-1, y=-3.

63
EQUATIONS SYSTEMS WORD PROBLEMS

1. Find the value of two numbers if their sum is 12 and their difference is 4. [Sol: 4 and 8.]

2. The difference of two numbers is 3. Their sum is 13. Find the numbers. [Sol: 5 and 8.]

3. A hotel has double rooms and single rooms. In total there are 90 rooms and 165 beds. How
many rooms are of each type? [Sol: 75 double rooms and 15 single rooms]

4. The sum of two numbers is 30. Three times the first number added to twice the second
number is 64. Find the numbers. [Sol: the numbers are 4 and 26]

5. A new hospital in Springdale starts out with 11 junior residents and 5 senior residents on its
staff. Management plans to hire additional personnel at a rate of 1 junior resident per
month and 4 senior residents per month. Eventually, there will be an equal number of each
on the hospital staff. How many of each type will there be? How long will that take?

6. A school canteen has tables that seat 6 people and tables that seat 4 people. There are 20
tables in total, and together they offer enough seating for 104 students. How many tables
of each type are there? [Sol: 12 tables for 6 people and 8 tables for 4 people]

7. Two co-workers (compañeros de trabajo) decided to invest in two companies at the same
time, one that produces software and one that does biotechnology research. Juan
purchased (compró) 211 shares (acciones) in the software company and 494 shares in the
biotech firm, which cost a total of €50,445. At the same time, Alberto invested a total of
€23,903 in 323 shares in the software company and 189 shares in the biotech firm. How
much did each share cost? [Sol: Software €19, Biotech €94]

8. Manuela is going to send some flowers to his wife. Brookfield Florist charges £1 per rose,
plus £20 for the vase (jarrón). Craig's Flowers, in contrast, charges £2 per rose and £10 for
the vase. If Manuela orders the bouquet with a certain number of roses, the cost will be
the same with either flower shop. What would the total cost be? [Sol: £30 for 10 roses]

9. Two co-workers (compañeros de trabajo) picked up (cogieron) some writing instruments at


the office supply store. Alexander selected 8 boxes of pencils and 5 boxes of ballpoint pens,
paying €31. Next, Jack spent €40 on 5 boxes of pencils and 10 boxes of ballpoint pens. How
much does a box of each cost? [Sol: Pencil: 2€, ballpoint: 3€]

10. Pedro and Vicente went to an arcade where the machines took tokens
(fichas). Pedro played 9 games of skee ball and 3 games of pinball, using a
total of 27 tokens. At the same time, Vicente played 2 games of
skee ball and 3 games of pinball, using up 13 tokens. How many
tokens does each game require? [Sol: Skee ball: 2; pinball: 3]

64
11. A TV station executive is planning the new lineup for next season's shows. On Monday
nights, there will be 3 sitcoms and 4 dramas, for a total of 230 minutes of programming,
not counting commercials (anuncios). On Tuesday nights, he has scheduled (programado)
2 sitcoms (comedia de situación) and 4 dramas, for a total of 208 minutes of non-
commercial programming. All sitcoms have the same length and all dramas have the same
length. How long is each type of show? [Sol: Sitcom 22m; Drama 41m]

12. Jackie and Sofía have summer jobs selling newspaper subscriptions door-to-door, but their
compensation plans are different. Alejandro earns a base wage of €15 per hour, as well as
€1 for every subscription that she sells. Sofía gets €4 per subscription sold, in addition to a
base wage of €6 per hour. They will end up earning the same amount if they each sell a
certain number of subscriptions. How much would each one earn? How many subscriptions
would that be? [Sol: €18 for 3 subscriptions]

13. Two car owners are in need of car repairs. Jim will need to pay the mechanic €1 per minute
for labor, plus €400 to cover the cost of new parts. Martin will need to pay €200 for parts
and €3 per minute for labor. Depending on how long each repair takes, the two jobs might
end up costing the same amount. How much time would that take? [Sol: €500 in 100min.]

14. Some History teachers at Hampton High School are purchasing tickets for students and
their adult chaperones to go on a field trip to a nearby museum. For her class, Mrs.
Jefferson bought 25 student tickets and 31 adult tickets, which cost a total of £901. Mr.
Jefferson spent £815, getting 29 student tickets and 25 adult tickets. What is the price for
each type of ticket? [Sol: Students £10, Adults £21]

15. Catalina and Moisés met for dinner in downtown (en el centro de) Kensington. Catalina did
flat-rate valet parking for €19. Moisés went to another valet and paid €7 up front and €4
for every hour, including the first hour. Ultimately, the friends ended up paying the same
amount. How much did each one pay? How long did they stay? [Sol: 19€, 3 hours]

16. Sabrina has been planting young trees in her garden. The maple (arce) tree that is 40
centimeters tall is growing 4 centimeters per month, whereas (mientras que) the oak
(roble) tree that is 30 centimeters tall is growing 5 centimeters per month. In a few months,
the two trees will be the same height. What will that height be? [Sol: 80cm in 10 months]

17. Britney is a costume designer for the local children's theater company. Yesterday, she
sewed (coser) 1 female costume and 4 male costumes, which used 49 meters of fabric.
Today she sewed 2 female costumes and 1 male costume, which used a total of 42 meters.
How many meters of fabric does each type of costume require? [Sol: 17 female, 8 male]

65
18. A couple of friends decide to race (echar una carrera) each other. Dylan can run 4 meters
per second, whereas Monique can run 9 meters per second. Because he is slower, Dylan
also gets a head start of 20 meters. Shortly after they start running, Monique will catch up
to Dylan. How far will Monique have to run? How long will that take? [Sol: 36m in 4 sec.]

19. Last Wednesday, two friends met up after school to read the book they were both assigned
in Literature class. Leo can read 3 pages per minute, and he had already read 50 pages. Eva,
who has a reading speed of 4 pages per minute, had read 40 pages. Eventually they had
read the same number of pages. How many pages had each of them read? [Sol: 10 m, 80 p.]

20. Mrs. Rivers sometimes makes her melon salad for special events.
When she made it a couple months ago, she used 3 kilograms of
honeydew melon (es un tipo de melón) and 1 kilogram of watermelon
(sandía), which cost her €6. Today, she used 2 kilograms of honeydew
melon and 2 kilograms of watermelon, spending a total of €8 on the
melons. Assuming that the prices of the melons haven't changed, how much does a
kilogram of each type of melon cost? [Sol: Honeydew 1€; watermelon €3.]

21. Mrs. Faulkner, the P.E. teacher, is pairing off students to race against each other. Hannah
can run 6 meters per second, and Lara can run 8 meters per second. Mrs. Faulkner decides
to give Hannah a head start of 14 meters since she runs more slowly. Once the students
start running, Lara will quickly catch up to Hannah. How far will Lara have to run? How long
will that take? [Sol: 56m, in 7 seconds]

22. Joe and Raymond each opened a savings account on the same day. Joe started by putting
£30 in his account, and he will deposit an additional £1 each week. Raymond made an
initial deposit of £10, and he will add £2 more each week. Eventually, Joe and Raymond will
each have the same amount saved.
What is that amount? How long will take them? [Sol: £50 in 20 weeks]

23. Wang's Bakery (panadería) sold one customer 2 dozen chocolate cookies and 6 dozen
oatmeal (harina de avena) cookies for £60. The bakery also sold another customer 2 dozen
chocolate cookies and 1 dozen oatmeal cookies for £25. How much do the cookies cost?
[Sol: Oatmeal £0.58, Chocolate £0.76]

24. Our school is selling tickets to a choral performance. On the first day of ticket sales the
school sold 3 senior citizen tickets and 1 child ticket for a total of $38. The school took in
$52 on the second day by selling 3 senior citizen tickets and 2 child tickets. Find the price of
a senior citizen ticket and the price of a child ticket. [Sol: senior: $8, child: $14.]

66
25. Last Wednesday, two friends met up after school to read the book they were both assigned
in Literature class. Tony can read 1 page per minute, and he had already read 60 pages.
Belle, who has a reading speed of 2 pages per minute, had read 20 pages. Eventually they
had read the same number of pages.
How many pages had each of them read? How long has taken them? [100pages in 40m]

26. The state fair is a popular field trip destination. This year the senior class at High School A
and the senior class at High School B both planned trips there. Senior class at High School A
rented and filled 8 vans and 8 buses with 240 students. High School B rented and filled 4
vans and 1 bus with 54 students. Every van had the same number of students in it as did
the buses. Find the number of students in each van and in each bus. [Sol: Van: 8, Bus: 22.]

27. Brenda's school is selling tickets to a spring musical. On the first day of ticket sales the
school sold 3 senior citizen tickets and 9 child tickets for a total of $75. The school took in
$67 on the second day by selling 8 senior citizen tickets and 5 child tickets. What is the
price each of one senior citizen ticket and one child ticket? [Sol: senior: $4, child: $7.]

28. The senior classes at High School A and High School B planned separate trips to New York
City. The senior class at High School A rented and filled 1 van and 6 buses with 372
students. High School B rented and filled 4 vans and 12 buses with 780 students. Each van
and each bus carried the same number of students. How many students can a van carry?
How many students can a bus carry? [Sol: Van: 18, Bus: 59.]

29. Pedro and Ana are selling flower bulbs (bulbo) for a school fundraiser (recaudar fondos).
Customers can buy bags of windflower (anémona) bulbs and bags of daffodil (narciso)
bulbs. Pedro sold 10 bags of windflower bulbs and 12 bags of daffodil bulbs for a total of
$380. Ana sold 6 bags of windflower bulbs and 8 bags of daffodil bulbs for a total of $244.
What is the cost each of one bag of windflower bulbs and one bag of daffodil bulbs?
[Sol: bag of windflower bulbs: $14, bag of daffodil bulbs: $20.]

67
2º ESO Mathematics Exam (trial exam) IES Extremadura

Bilingual section

SYSTEMS OF EQUATIONS.

1 (1 pt) Fill, so the solution is x=3, y=-2

2 (2 pts.) Solve using the addition method:

3 (1.5 pt.) Solve using the algebraic equation method:

4 (1.5 pt.) Solve using the graphing method:

5 (2 pts.) Solve using substitution:

6 (2 pts.) Mary’s age is double than Julia’s age, and if we add their ages of ten years ago, we
obtain Mary’s current age. What is the age of each Mary and Julia?

68
9. FUNCIONES
 EL PLANO CARTESIANO

Para dar coordenadas a los puntos de un plano usamos dos rectas, que se llaman ejes de
coordenadas. Su punto de corte se llama origen de coordenadas (O).
y
5
- Eje de abscisas (X): recta horizontal.
4

- Eje de ordenadas (Y): recta vertical. II Segundo 3 I Primer


Cuadrante 2
Cuadrante
El plano queda dividido en cuatro cuadrantes. 1
x

-5 -4 -3 -2 -1 1 2 3 4 5
-1
Para cada punto, escribimos sus coordenadas entre -2
III Tercer IV Cuarto
paréntesis. Ejemplo P(3,2). -3
Cuadrante Cuadrante
-4

-5
 FUNCIONES

- Una función es una relación entre dos conjuntos (inicial y final), de manera que a cada
elemento del primero le corresponde uno solo del segundo.
- Variable independiente (x): los valores del conjunto inicial.
- Variable dependiente (y): los valores del conjunto final.

Formas de expresar una función:


- Lenguaje natural: una frase que describe la relación. Ejemplo: “el doble de cada número”.
- Tabla de valores: en una fila (o columna) colocamos la variable independiente, y en otra la
dependiente. Ejemplo: x 0 1 2 3 4 5
y

y 0 2 4 6 8 10 5
4
3

- Una gráfica: en el eje de abscisas colocamos la variable 2


1
independiente (x) y en el de ordenadas la dependiente (y). -5 -4 -3 -2 -1 1 2 3 4 5
x

-1

Ejemplo: -2
-3
-4
-5

- Una fórmula: una ecuación describe cómo están relacionadas las variables. Substituyendo el
valor de “x” en la fórmula, se obtiene el de “y”. Ejemplo: y=2x. Para x=3 resulta y=2·3=6.

69
Ejemplos de funciones:
- Proporcionalidad directa (y=mx): En el cine, Dinero = 4€ nº de entradas.
- Función afín (y=mx+n): Al llamar por teléfono, Precio=0.06·minutos + 0.12
Su gráfica es una recta. El número m es la pendiente, y mide la inclinación de la recta.

- Proporcionalidad inversa . Al comprar botes con 30€,

Características de una función:


- Continua: puede dibujarse de un solo trazo.
- Discontinua: si la gráfica da algún salto.

- Puntos de corte con los ejes: los que están sobre los ejes de coordenadas.

- Creciente: cuando al que al aumentar “x”, aumenta “y”.


- Decreciente: cuando al aumentar “x”, disminuye “y”.

- Máximo: punto de la gráfica donde la función toma el valor más alto de su entorno.
- Mínimo: punto de la gráfica donde la función toma el valor más bajo de su entorno.

70
FUNCTIONS

 CARTESIAN PLANE

We use a pair or lines to set coordinates for each point of a plane; they are called coordinate
axis. They meet at the origin of coordinates (O).
y
5
- Abscissa axis (X): horizontal line.
4

- Ordinate axis (Y): vertical line. II Second 3 I First


Quadrant 2
Quadrant
The plane is divided into four regions called quadrants. 1
x

-5 -4 -3 -2 -1 1 2 3 4 5
-1
For each point we write its coordinates in parentheses. -2
III Third IV Fourth
Example: P(3,2). -3
Quadrant Quadrant
-4

 FUNCTIONS -5

- A function is a relationship between two sets (initial/input and final/output), such that each
input is associated to one and only one output.
- Independent variable (x): the input to a function.
- Dependent variable (y): the output.

Ways to give a function:


- Natural language: a sentence describing the relationship. Example: “the double of each
number”.
- Table of values: we put the independent variable in a row (or column) and the dependent in
another. Example: x 0 1 2 3 4 5
y 0 2 4 6 8 10
y

5
4
- A graph: we put the independent variable (x) in the 3

abscissa axis and the dependent (y) in the ordinate. 2


1
x

Example: -5 -4 -3 -2 -1
-1
1 2 3 4 5

-2
-3
-4
-5

- A formula: and equation describing how the variables are related. By substituting the value
of “x” into the formula, one get “y” value. Example: y=2x. For x=3 results y=2·3=6.

71
Examples of functions:
- Direct proportionality (y=mx): At the cinema, Money = 4€ no of tickets.
- Affine function (y=mx+n): When phoning, Price=0.06·minutes + 0.12
Their graph is a line. Number m is the slope, and measures the steepness of the line.

- Inverse proportionality . Buying cans with 30€,

Characteristics of a function:
- Continuous: it can be drawn without lifting the pencil from the paper (no gaps).
- Discontinuous: its graph jumps somewhere (there are gaps).

- X and Y Intercepts: the points where the graph meets the axis (X e Y).

- Increasing: if the more value of “x”, the more value of “y”.


- Decreasing: if the more value of “x”, the less value of “y”.

- Maximum: point where the function takes the largest value within a neighbourhood.
- Minimum: point where the function takes the smallest value within a neighbourhood.

72
LINEAR FUNCTIONS WORD PROBLEMS

Complete the following information for the following functions before answering:
a) Dependent variable (y) and independent variable (x).
b) Algebraic expression using a formula. Say what is the slope and ordinate at the origin.
c) Table of values.
d) Graph.

1. Juan has written 32 pages for an essay. From now on, he estimates is going to write two
pages per hour. Variables: hours and number of pages written.
How long will Juan have to spend writing this week in order to have written a total of 46
pages?
2. Pipo the puppy weighs 1kg. Each month it gains 4kg. Variables: weight and months.
How heavy is Pipo at 7 months old?
3. Ana the florist has got 92 carnations (claveles) at her flower shop. She uses 11 carnations
with each arrangement she puts together. Variables: carnations left and arrangements.
What is the greatest number of arrangements Ana can put together if she wants to have
40 carnations left?
4. Carlos has got 93 pears at his greengrocer’s. He is assembling gift basket and uses 6 pears
on each. Variables: pears left and baskets.
If Carlos assembles 10 gift baskets, how many pears will he have left?
5. We are screen-printing (serigrafiar) a batch (conjunto) of shirts. The printer takes 5
minutes to get ready and 2 minutes per shirt. Variables: number of shirts and minutes.
If it takes 41 minutes to screen-print a batch of shirts, how many shirts are in the batch?
6. Ana has 68g of jelly (mermelada) left. If she uses 9g on each sandwich she makes.
Variables: jelly left and no. of sandwiches.
Up to how many sandwiches can Ana make and still have 32g of jelly left?
7. Pedro buys 4 souvenirs each day he spends on vacation (estar de vacaciones).
Variables: number of souvenirs and days of vacation.
After 5 days of vacation, how many total souvenirs will he have bought?

73
8. It costs $12 to attend (asistir) a golf clinic (clases de golf) with a
local pro (instructor). Buckets of balls for practice during the
clinic cost $3 each. Variables: buckets and dollars.
How many buckets can you buy at the clinic if you have $30 to
spend?

9. Paulo has $145 in his savings account. He earns $36 a week mowing lawns (cortar el
césped). Variables: weeks and dollars.
If Paulo saves all of his earnings, after how many weeks will he have $433 saved?

10. A survey of teens showed that teens in Pittsburgh aged 12-17 spend 15.8 hours per week
online. Variables: hours and weeks.
In how many weeks will a teenager spend 237h online?

11. An online retailer (minorista) charges €6.99 plus €0.55 per pound (1p=453.6g) to ship
electronics purchases. Variables: pounds and euros.
How many pounds is a DVD player for which the shipping charge is €11.94?

12. Caitlin has a $10 gift certificate to the music store. She has chosen a number of CDs from
the $7 bargain bin. Variables: dollars and CDs.
If the cost of the CDs is $32 after the gift certificate is credited, how many CDs did she buy?
13. Mrs. Jackson earned a $500 bonus for signing a one-year contract to work as a nurse. Her
salary is $22 per hour. Variables: hours and dollars.
If her first week’s check including the bonus is $1,204, how many hours did Mrs. Jackson
work? [Sol: 32 hours]
14. Morgan subscribes to a website for processing her digital pictures. The subscription is
€5.95 per month and “10 by 15cm” prints are €0.19 each. Variables: prints and Euros.
How many prints did Morgan purchase if the charge for January was €15.83?

74
FUNCTIONS – PRACTICE

For the following functions, make a plot using geogebra or wiris. Then describe them. Use a
chart as the one showed below. Points

Discontinuity
1. Ana has been analyzing the light bill for some
Extremes Maxima
months. She has found that it follows the equation
Minima
y=10x2-60x+130, from the first month to the 6th
Intercepts y-intercept
month.
x-intercepts
a) When did she spend the most? And the least? Intervals

b) How much were each? Monotony increasing

decreasing

constant
2. In a restaurant, they are using a model to predict 32
y f(x)=x^3-9x^2+15x+25
f(x)=x^3-9x^2+15x+25

30

how many clients they will have, in the next 6 28

hours. The equation is y=x3-9x2+15x+25.


26

24

22

a) When are they expecting to have the most


20

18

clients? How many will there be?


16

14

12

b) And the least clients? 10

c) Describe the function. 4

2
x
-2 -1 1 2 3 4 5 6 7
-2
3. A rocket is blasting off in a minute. Considering the working of the engines, the equations
of its height as a function of the time, from minute 1 to 10, will be y=2x2-32x+30+60 ln(x).

4. Analysts of a company have predicted that the shares (acciones) price in the next 20 days
can be predicted using the equation .

When will the price be higher? and lower?


y
140
5. The city council (ayuntamiento) has ordered a sculpture. Its
120
price will depend on its size (raw material pieces, working
hours, etc.). Up to 5 metres, the price is given by the 100

equation y=-x4+12x3-40x2+48x. 80

60
a) What is the highest price?
40

b) Describe the function. 20

-10 -9 -8 -7 -6 -5 -4 -3 -2 -1 1 2 3 4 5 6 7 8 9 10

-20

-40

-60

75
26 y f(x)=4(1/x+1)
f(x)=(x+1)(x-3)
24 f(x)=4x-10

22
y f(x)=(x^2+13x-28)/10
10 f(x)=5-2^(x-4)
20
f(x)=x^(1/3)

6. Describe the following functions, considering18 their graph


8
36 y

6 16 33
30
a) 4 b) 14
c) 27
24
12 21
2
10 18
x 15
-20 -18 -16 -14 -12 -10 -8 -6 -4 -2 2 4 6 8 10 12 14 16 18 20 22 24 8 12
9
-2 6 6
3 x
-4 4
-1.5 -1 -0.5-3 0.5 1 1.5 2 2.5 3 3.5 4 4.5
-6 2 -6
-9 x
-8 -12 -11 -10 -9 -8 -7 -6 -5 -4 -3 -2 -1 1 2 3 4 5 6 7 8 9 10-1211 12 13 14 15
-2 -15
-10 -18
-4 -21
-12 -24
-6 -27
-30
d) -14
e) -8 f)
-10
-12

g) h) i)

k) l) m)

7. Plot and describe these functions (use geogebra or wiris, for e.g.).
a) f(x)=x6-11x5+39x4-45x3. b) f(x)=x6-5x5+8x4-4x3 c) f(x)=x5-7x4+12x3.

 3x 3  27 x x 3  5x 2  7 x  3 10 x  20
d) f ( x)  e) f ( x)  f) f ( x)  2
3x 2  5 x 2
x  x2

76
2º ESO Mathematics Exam (trial exam) 7 y IES Extremadura
6
Bilingual section
5
FUNCTIONS. 4
3
1 (2.25 pts.) Say if they are a function and explain why:
2
a) x y b) x y c) 1
2 3 1 7
-9 -8 -7 -6 -5 -4 -3 -2 -1 1 2 3 4 5 6 7 8 9
4 2 4 2 -1
5 1 3 5 -2
2 6 5 1 -3
1 9 2 9 -4
-5
2 (1 pt. ). Plot the points and name the quadrant or axis in which they lie. -6
Point Lies in... Point Lies in... -7
P1(4,3) P5(0,-2)
P2(1,1) P6(1,-2)
P3(-2,4) P7(4,2)
P4(-3,0)

3. (3 pts.) Fill the chart, and graph the functions. (a) (b) (c)
x y x y x y
Ordinate at
Equation Slope
the origin
y= 2x – 3
y=-x+1
3 -2

4 (2.25 pts.) Write the characteristics of this function:

 

5 (1.5 pts.) Juani’s got 80 soup spoons left at her restaurant. She uses 4 spoons each time she sets a
table.
a) Relate no. of spoons left and tables set, including formula, slope and ordinate at the origin.
b) How many tables has she set if there are 28 spoons left?

77
10. ESTADÍSTICA
- La estadística es la ciencia que se ocupa de recoger, resumir y analizar datos. También de
sacar conclusiones, hacer predicciones y tomar decisiones.
- Población: los elementos de los que queremos obtener información.
- Muestra: parte de la población que usamos en el estudio. Censo: se toma toda la población.
Muestra sesgada: Si algún elemento tiene más probabilidad de ser elegido.
- Variable/carácter estadístico: característica de la población en la que estamos interesados.
Cuantitativas: se miden con números.
Discreta: puede tomar pocos valores. Ejemplo: “nota de la 1ª evaluación”.
Continua: puede tomar muchos valores. Ejemplo: “altura”.
Cualitativas: medimos cualidades (y no números). Ejemplo: “color favorito”.

RESÚMENES DE DATOS
 Tabla de Frecuencias
- Frecuencia absoluta: número de veces que se repite el dato.
- Frecuencia relativa: indica qué parte del total es ese dato. Podemos medirlo en porcentaje.
- Frecuencia acumulada: el número de datos que hay menores o iguales que él.
Si la variable es continua, hay que agrupar los datos por intervalos.

 Gráficos Estadísticos
- Diagrama de barras (y de puntos): dibujamos la variable estadística en el eje horizontal y la
frecuencia en el vertical. Con datos agrupados, usamos un histograma (barras juntas).
- Polígono de frecuencias: unimos las barras del gráfico con segmentos.
- Diagrama de sectores: dividimos un círculo en partes proporcionales a la frecuencia.
- Caja y bigotes: ponemos los valores mínimo, máximo y cuartiles en la recta numérica.
- Diagrama de tallo-hojas: ordenamos los datos en una tabla. El tallo es la primera parte del
número y la hoja la última cifra. Hay una hoja por cada dato (y puede haber hojas iguales).

 Medidas de Centralización

- Media ( ): suma de los datos dividido entre el número de datos. Puede ser ponderada,
(multiplicando por unos “pesos”, para dar más importancia a algunos datos).
- Mediana: tras ordenar los datos, el que queda justo en la mitad. Los cuartiles son los que
quedan en el 25%, 50% (la mitad), y el 75%.
- Moda: el dato que tenga la mayor frecuencia (puede haber varias modas).

78
STATISTICS
- Statistics is the science that gathers, summarizes and analyses data. Also, it is dedicated to
find conclusions, make predictions and make decisions.
- Population: is a collection of elements whose properties are analyzed.
- Sample: is a part of the population of interest. Census: when we take the whole population.
Biased sample: some people or items are more likely to be chosen than others.
- Variable/characteristic: feature of the population we are interested in. It can be
Quantitative: anything that can be expressed as a number.
Discrete: it can only take a few values. Example: “mark in the 1st term”.
Continuous: it can only take many values. Example: “height”.
Qualitative: when we measure qualities (and not numbers). Example: “favourite colour”.

DATA SUMMARIES
 Frequency tables/charts
- Absolute frequency: the number of times the data value occurs.
- Relative frequency: what part of the total is that data. It can be measured in percentage.
- Cumulative frequency: the number of data less or equal than a given data.
If the variable is continuous, is convenient to group data into intervals.

 Statistic Graphs
- Bar graph: (and line plot) we put the statistic variable on a horizontal axis. Frequency is put
on the vertical. If data are grouped, we use a histogram (bars are drawn together).
- Frequency polygon: join the bars by line segments.
- Circle graph (pie chart): a circular chart divided into sectors, proportional to the frequency.
- Box and whisker: we place the least data, the greatest data and quartiles on the number line.
- Stem-and-leaf plot: the stem tells the first part of the number and the leaf tells the last digit
of the number. There is one leaf for each data (and leaves can be repeated).

 Central Tendency measures


- Mean ( ): the sum of the numbers divided by the number of data. It can be weighted,
(multiplying by some “weights”, to give more importance to some data).
- Median: when the numbers are arranged from least to greatest, it is in the middle.
Quartiles split the data into four sections. Each section contains 25% of the data.
- Mode: the number that appears most often. (There can be several modes.)

79
STATISTICS - WORD PROBLEMS
1. Describe the following statistics and say if the samples are biased.

a) In order to check the quality of the restaurants/meals in her region, Juana tried 2 meals
from each town.
b) For a statistics project, Pedro is surveying the favourite singer of his 30 classmates. So he
distributed a questionnaire and asked them to fill it out. Only her 10 best friends did.
c) Marta noted the price of 32 randomly chosen books from each library in his city.
d) To learn the amount of hours people spend to prepare a dance competition, Paula
questioned the top 2 contestants in the competition.
e) Miguel questioned the first 22 diners (comensal) to arrive at the restaurant about their
opinion (OK, not bad, disappointing,...) about the time they have to wait to be seated.
f) Luisa polls (encuestar, sondear) 47 pet owners in the city about how much they spend on
their pets monthly.
g) Alicia wants to know the opinion of the patients about the hospital she works in (bad,
good, excellent, ...). She surveys the 5 healthiest patients.

2. Describe the following surveys


a) In a given neighbourhood (vecindario), the owners of 50 dogs were asked about the
height and breed (raza) of their pets, and the number of times they walked them each
week.
b) Of all the people who are members at a sports centre, 30 people are asked how much
time they spend exercising.
c) Of all the people entering in an airport, two of every 15 people are asked which
destination they have chosen for their holidays.
d) Twenty of a school’s students are chosen at random and are asked about the number
of times they go to the cinema in a month. We picked 2 from each of the 10 classes.
e) We ask one out of every 15 customers on a video shop about the type of film he has
hired (alquilar).
Sol: a) Pop: the dogs in the neighbourhood. Sample: 50 dogs. Var: nº. of times (discrete), height (continuous),
breed (qualitative).
b) Pop: members of the sport centre. Sample: 30 people. Var: time spent exercising (continuous).
c) Pop: people in the airport. Sample: 2 of every 15. Var: destination (quatitative).
d) Pop: all the students at the school. Sample: 20 students. Var: no. times gone to the cinema (discrete).
e) Pop: customers of the video-club. Sample: 1 out of 15. Var: type of film hired (qualitative).

80
3. Mr. Marshall keeps track of the number of hours his 6 students spent doing volunteer
work. His records show:
7.5 hours 5.4 hours 8.6 hours

5.4 hours 5.4 hours 3.1 hours

What are the mean and mode hours spent doing volunteer work? [Sol: Mean: 5.9h. Mode: 5.4h]

4. Calculate the final mark of these students, considering their marks and how much each
subject (asignatura) is worth (valorada). [Hint: Use a weighted mean]

Pupil Maths Science English Mark a) Maths is worth “2”, Science “1” and English “2”
Luis 7 9 5
b) Maths is worth “1”, Science “1.5” and English “2”
Juana 8 6 5
Eva 6 10 7 c) Maths is worth “2.5”, Science “2” and English “1”

5. A candidate obtains the following percentages in an examination. English 46%,


Mathematics 67%, Science 72%, Ph.E. 58%, Technology 53%. It is agreed to give double
weights to marks in English and Mathematics as compared to other subjects. What is the
weighted mean? [Sol: 58.43%]

6. The English teacher uses a weighted mean to calculate the final mark of her pupils. The
chart shows the marks of 4 pupils (out of 10) in each section, and the weights in
percentage. Calculate their final marks.
Writing Listening Reading Exercises Behaviour Final
Pupil
(30%) (25%) (15%) (20%) (10%) Mark
Sergio 8 7 7 10 10 =8,20
Elena 10 9 10 5 5 =8,25
Isabel 6 7 5 9 10 =7,10
Alonso 5 5,5 6,5 4 2 =4,85

7. In a school, 85 boys and 35 girls appeared in a public examination. The mean mark of boys
was found to be 40% whereas (mientras que) the mean marks of girls was 60%. Determine
the average marks percentage of the school. [Hint: Use a weighted mean] [Sol: 45.83%]

8. A class of 25 students took a science test. 10 students had an average (arithmetic mean)
score of 80. The other students had an average score of 60. What is the average score of
the whole class? [Hint: Use a weighted mean] [Sol: 68]

9. A car dealer has got the following information about the weekly car sales:
- They are usually between 10 and 20.
- A 25% of the weeks is less than 12 and a 25% is more than 18.
- Half of the weeks, the car sales are less than 16, and the other half, more than 16.
Draw a box-and-whisker plot using this information.

81
10. The data below shows the ages of the people who subscribe to two different magazines.
Magazine 1 20 21 32 19 47 65 34 21 33 52 24 20 19 31 23 22
Magazine 2 45 67 20 72 54 37 51 54 50 52 44 39 85 29 57 60

a) Find the subscribers-mean-age of each magazine.


b) Draw a stem-and-leaf plot.
c) Draw a box-and-whisker plot. Use it to compare the ages of the readers.

11. These box-and-whisker plots show the ages of people using a public pool (piscina) in two
different days. Compare the differences in ages of the pool-users.
a) Looking at the minimum and maximum values.
b) Looking at the medians
c) Looking at the quartiles

12. These box-and-whisker plots show the prices of stock in two stores. What do they tell you
about the price differences in the two stores?
a) Looking at the minimum and maximum values.
b) Looking at the medians
c) Looking at the quartiles
[Sol: a) Store B has the lowest priced item ($10). Store A’s lowest price is much higher ($30). Both stores
sell their most expensive item for the same price ($90). So Store B has a greater range of prices.
b) Store A’s stock has a median price of $70, whereas (mientras que) Store B’s stock has a median
price of $40. So Store B’s items are typically less expensive than Store A’s. (Half of Store A’s stock is
under $70, but half of the Store B’s stock is under $40)
c) The middle 50% of the prices in Store B are much more spread out (esparcidos) than they are in
Store A. They go from $30 to nearly $70. The middle 50% of the prices in Store A are concentrated
more tightly (más concentrados) around the median value of $70.]

13. The box-and whisker plots below show the test scores in two classes. Compare them
a) Looking at the minimum and maximum values.
b) Looking at the medians
c) Looking at the quartiles
[Sol: a) Class A’s test scores had a much larger range than Class B’s. Both the highest and lowest scores
overall were found in Class A.
b) Class B’s scores were generally higher than Class A’s. The median score for Class B was more than
50, but for Class B it was about 43.
c) More than half the students in Class B scored more than 50, whereas in Class A only one-quarter
scored more than 50.

82
14. Daw and Eric, who own flower shops, compared their daily bouquet sales. These box-and-
whisker plot show the results:

What percent of the days sell each between 25 and 40 bouquets?


Compare the number of bouquets sold by each (min. and max. values, median and quartiles)

15. Look at this box-and-whisker plot:

Compare the wait time of both doctor’s offices (min. and max. values, median and quartiles).

16. We have got the following data on the number of apples per tree on a field:
60 26 69 73 26 44 40 35 49 60 24 61 26
Draw the corresponding stem-leaf plot.

17. Look at this stem-and-leaf plot:

How many students scored fewer than 66 points?

Calculate the mean.

Draw the box-and-whisker plot.

18. Look at this stem-and-leaf plot:

Calculate the mean Committee size

What is the size of the largest committee?

Draw the box-and-whisker plot.

83
19. This line plot shows the number of goals scored in the soccer matches played by a school
team this year.
a) In how many matches did the team score 0 goals?
b) In how many matches did the teams score 2 or more goals?
c) Draw a box-and-whisker plot and the frequency table.
d) Find the central tendency measures (mean, mode, median).

20. This pictograph shows the ages of cars parked in a parking lot (rounded to the nearest year).
a) How many cars are parked? [Sol: 20 cars]

b) Draw a box-and-whisker plot and the frequency table.


c) Find the central tendency measures.

21. Draw a stem-and-leaf plot for the following sets of data.


Class A-Test scores 6.5 6.9 7 7.2 7.8 8.1 8.5 8.7 9 9.8
Class B-Test scores 8.7 8.8 8.8 9 9.2 9.5 9.5 9.5 9.7 9.7
a) Draw a back-to-back stem-and leaf plot, and look at its shape to compare the data sets.
b) Group by units and make a table of values for each set.
[Sol: The scores of Class A are fairly evenly spread (igualmente repartidos) between 6.5 and 9.8. The
scores of Class B are clustered tightly (muy agrupados) around the high-8s and 9s].

22. Two holiday companies each organized a trip to visit pyramids in Egypt for people aged over
50. Their ages were
Company A 56 57 61 64 65 66 67 68 68 69 70
Company B 52 55 55 56 59 60 63 65 65 66 66
68 71 71 73 79 80 82 83 85
a) Draw a back-to-back stem-and-leaf plot for the data and use it to compare the ages of the
people who travelled with each company.
b) Group each set of data by tens and make the frequency table.

[Sol: The ages of the people who travelled with Company B were fairly evenly spread (repartidos
igualmente) between 52 and 85 (the leaves are all of similar length). The people who travelled with
Company A were typically younger, and were closer together in age –most of them were in their
60s- (This is shown by a very long leaf representing the people in their 60s]

23. We are conducting a survey about how many pupils pass 2ºESO in the bilingual sections at
Spain. We have got the following information from some high-schools.
Pupils passing 24 25 26 27 28 29 30
No. High-Schools 1 3 10 11 25 30 20

- Describe this survey; make the frequency table and graphs. Find mean, mode and median.

84
24. A factory is interested in the size of the bolts (tornillo-perno) they are producing. So they
pick some bolts from the production line and analize them. The results are the following:
Bolt size 5.45 5.46 5.47 5.48 5.49 5.50 5.51 5.52 5.53 5.54 5.55
No. of bolts 2 8 16 26 28 40 28 26 16 8 2

- Describe this survey and summarize the data using the frequency table, statistics graphs and
central tendency measures.

25. This line plot shows the test scores of 26 students.


Use it to make the frequency table, statistics graphs
and central tendency measures.

26. Draw a line plot to represent the prices of different


story books given below. Find the central tendency measures.
$10, $30, $10, $10, $20, $30, $50, $40, $45, $50, $10, $30, $35, $30, $20, $40, $50

27. This table shows the number of television sets owned by 30 different families in Mérida.
a) Describe this survey. Number of TVs
2 1 2 4 3 0
b) Make a line plot and box-and-whisker plot for the data. 2 3 2 3 4 2
1 2 2 3 4 0
c) How many televisions do most families own? 3 1 3 2 1 2
5 3 4 3 0 0
Which central tendency measure is that?
d) Compute all the other central tendency measures.
e) Draw a box-and-whisker plot. What is the range?

28. This table shows the number of times per day that students in “A”-high school go to their
lockers (taquillas). Student Locker Visits
2 2 0 1 2 2 3 4
a) Describe this survey. 0 5 2 5 2 5 2 4
2 4 6 4 5 6 5 6
b) Make a table of frequency of the data. 2 2 0 1 4 6 10 2
c) Make a line plot and the box-and-whisker plot for the data.
d) Find the central tendency measures.
e) Would the mean, median, or mode best represent the data? (Explain why the mean
does not best represent the data).
f) If the value 10 were dropped from the data, find the median and the mode of the
remaining data. Would the mean now best represent the data?

85
29. This table shows the number of books sold in Luisa’s book shop
each day for 20 days. Book sales per day
23 18 23 15
a) Describe the survey. 24 16 0 11
19 10 13 17
b) Make the frequency table and statistic graphs. 12 23 11 16
c) Find the mean, median, and mode of the data. 36 24 12 27

d) Which measure of central tendency would be misleading (engañosa) in describing the


book sales? Explain.

30. The following data set represents the NBA Team Payrolls (salarios) in millions of dollars.
24.1 24.9 25.2 25.5 25.9 27.0 27.1 27.1 27.3 27.4
27.8 27.8 28.0 28.4 28.5 28.5 32.1 34.1 34.2 34.6
36.6 36.7 38.9 40.9 42.1 42.9 45.8 56.5 61.8
a) Why is it better to use a histogram? Draw it (group by fives).
b) Make the box-and-whisker plot.

31. The following data set represents the Spending per Student in some schools.
3439 3660 3844 4037 4149 4261 4280 4519 4586 4611 4674 4697 4761 4894 4898
4915 4921 5049 5097 5107 5109 5114 5288 5516 5598 5630 5651 5659 5713 5720
5723 5751 5879 5893 5899 5971 6069 6263 6600 6621 6658 6717 6958 6959 6983
7333 8473 8882 9175 9677 10180

a) Why is it better to use a histogram? Draw it (group by thousands).


Distance Freq.
b) Make the box-and-whisker plot. [6.50,6.75) 4
[6.75,7.00) 11
32. The distances achieved by 42 long-jumpers are shown in the following [7.00,7.25) 8
[7.25,7.50) 12
table. Depict the data with a histogram. [7.50,7.75) 6
[7.75,8.00] 1

33. Look at the histogram, about the commute time.


a) How many commutes (desplazamiento al trabajo)
are there in all?
b) Which range occurs most frequently?
And least?
c) Create the table of frequencies.

34. A machine at the candy factory dispensed different numbers Lemon-flavored candies per bag
of lemon-flavoured candies into various bags. Lemon-flavored Number of
candies bags
a) Create the table of frequencies. 0-19 60
20-39 10
b) Draw the histogram. Why is it better than a bar graph?
40-59 70
60-79 60
80-99 50
100-119 60
86
2º ESO Mathematics Trial Exam IES Extremadura
Bilingual section
Surname _________________________________________________ Mark
Name __________________________________

STATISTICS
1. (1.5 pts.) Analyse the following statistics studies (say the population, variable –and type-, and
sample). Also say if we a census was conducted.
a. At the greengrocer’s we are given an apple and they ask us to value its flavour from 1 to 10
in order to get an idea of how their apples are.
Population: Variable (and type):
Sample: Census? YES | NO

b. An advertisement/trailer shows 30 seconds of a film, so you can have an opinion about


the film (good, bad, very good or very bad).
Population: Variable (and type):
Sample: Census? YES | NO

c. Before the elections at Mérida, we ask one out of each 100 people for which party they
intend to vote.
Population: Variable (and type):
Sample: Census? YES | NO

2. (3.5 pts.). This plot compares the time spent by a couple of pupils each day studying for a statistics
exam.
Time spent studying (minutes)
Emilia Isabel
L_ _ _ S_ _ _ L_ _ _
8 6 5 2
9 8 5 5 3 3 0 3 0 5 6
5 4 0 0 4 0 8
5 0 0 5
6 4 6 6 8
Key: 2|1 means 21

a) (0.5 pts.) What is the name of this plot in English?

b) (0.5 pts.) Who has studied the most days? How many days?

c) (1 pt.) Make a box-and-whisker plot.

d) (1.5 pts.) Compare both pupils (looking at the maximum and minimum values, at the median
and at the quartiles). It is compulsory (obligatorio) to use sentences.

87
3. (4 pts.) We are surveying which means of transport pupils use at our high school. Thus, we ask the
first 20 pupils entering the High-School. We have got the following data:

Luis: School Bus Pedro: Bus Antonio: Car Ángel: Walking.


Laura: School Bus Ester: Walking Alex: Walking Irene: Walking.
Silvia: Walking Juan: Walking Javier: Walking Manuel: School Bus.
José: School Bus Carlos: Car María: Walking Concepción: Bus.
Clara: School Bus Ana: Walking Elena: Walking Martín: School Bus.

a) (0.5 pts.) Explain, using a sentence whether this sample is biased or not.

b) (1 pt.) Fill the frequency table.

c) (0.75 pts.) What is the mode of these data? (Say why)

d) (1 pt.) Is it better to use a bar graph or a histogram? Draw it along with the frequency polygon.

e) (0.75 pts.) Draw the pie chart.

4. (1 pt.) Find the Final Marks (out of ten), considering the mark on each exam and how much each
exam is worth. [Calcula las notas finales, considerando la nota de cada examen, y cuánto vale cada uno].

Exam#1 Exam#2 Exam#3


Pupil Mark
(1 pt.) (4 pts.) (3 pts.)
Pedro 10 7 9
Lucía 5 9 8
Eulalia 7 6 8

88
11. CUERPOS GEOMÉTRICOS Y VOLÚMENES
- Poliedro: cuerpo geométrico cerrado con caras planas. Arista: cada lado de las caras.
Poliedro convexo: si el segmento que une dos puntos cualquiera está dentro del poliedro.
Poliedro cóncavo: si alguno de los segmentos sale fuera del poliedro.
 Fórmula de Euler: en todo poliedro convexo (y todo cuerpo del “tipo” de la esfera)

Vértices – Aristas + Caras = 2.

 Poliedro regular (sólidos platónicos): si todas las caras son polígonos regulares iguales y en
cada vértice concurre el mismo número de caras. Hay cinco.

Tetraedro Hexaedro (cubo) Octaedro Dodecaedro Icosaedro

 Prisma: tiene dos caras iguales y paralelas “bases”, y las otras son paralelogramos.

Volumen: V=Abase·h.

Cuando la base es un paralelogramo, se llama paralelepípedo. Si además sus


ángulos son rectos, se llama ortoedro (como un rectángulo, con volumen).
 Pirámide: una cara es un polígono y el resto triángulos que concurren en un punto.

Volumen: V= Abase·h.

 Cuerpo de revolución: el obtenido haciendo girar una curva plana (generatriz) alrededor de
una recta (eje). Ejemplos: cilindro, cono y esfera.

Principio de Cavalieri: si dos cuerpos de la misma altura tienen secciones de


áreas iguales, entonces tienen el mismo volumen.

- Ejemplos: Volumen del cilindro: V=Abase·h. Volumen del cono: Abase·h.

Esfera: A=4r2. V= r3. Para una zona o un casquete, A=2r·h.

89
GEOMETRIC BODIES AND VOLUMES
- Polyhedron: a solid figure bounded by polygonal faces. Edges: sides of the faces.
Convex polyhedron: if any line segment joining two points is inside the polyhedron.
Concave polyhedron: if there is a line segment joining two points outside the polyhedron.
 Euler’s formula: in any convex polyhedron (or geometric body of the “type” of the sphere)

Vertices – Edges + Faces = 2.

 Regular polyhedra (platonic solids): when all its faces are equal regular polygons and on
every vertex meet the same number of faces. There are five.

Tetrahedron Hexahedron (cube) Octahedron Dodecahedron Icosahedron

 Prism: has two equal parallel faces “bases”, and the others are parallelograms.

Volume: V=Abase·h.

When the base is a parallelogram, it is called parallelepiped. Moreover if it is


right-angled, it is called cuboid (like a rectangle, with volume).
 Pyramid: one face is a polygon and the others triangles that meet at one point

Volume: V= Abase·h.

 Solid of revolution: it is obtained by rotating a plane curve (generatrix) around a straight


line (axis). Examples: cylinder, cone y sphere.

Cavalieri’s principle: if two bodies have the same height and their sections
have the same areas, then they have the same volume.

- Examples: Volume of the cylinder: V=Abase·h. Volume of the cone: Abase·h.

Sphere: A=4r2. V= r3. For any zone or cap, A=2r·h.

90
GEOMETRIC BODIES AND VOLUMES - PRACTICE

1. Find the surface area, using the nets.

a) b) c) d)

11.66cm
h
12cm

2 2 2 2
Sol: S=384 in S=687cm S=61.23ft S=4·60+144=384cm

2. Find the volume of each prism

a) b) c) d) e)

Sol: V=1496in3 V=910ft3 V=121cm3 V=1280cm3 V=242.25in3.

f) g) h) i) j)

3 3 3 3 3
Sol: V=136.71m V=86.645m V=1080in V=455m V=210m .

3. Find the volume of these figures

a) b) c) d) e)

Sol: V=277.1in3 V=4042.0m3 V=6308.3mm3 V=377in3 V=233m3.

f) g) h) i)

3 3 3 3
Sol: V=613.3cm V=47.4ft V=140.6m V=2364ft .

91
4. Work out the surface area and the volume of these shapes.

a) b) c) d)

Sol: V=240in3; S=236in2 V=420m3; S=568m2 V=21.2ft3; S=42.4ft2 V=31.4in3; S=69.1in2.

e) f) g) h)

Sol: V=28.3yd3; S=62.8yd2 V=257cm3, S=356cm2 V=7yd3; S=27yd2 V=120cm3; S=164cm2.

i) j) k) l)

Sol: V=5184in3; S=1872in2 V=13980yd3; S=3699yd2 V=5634in3; S=1292in2 V=1815.8ft3;S=917.3ft2.

l) m) n) o)

Sol: V=135744in3; S=17616in2 V=1566cm3; S=864cm2 V=300in3; S=320in2 V=254.8yd3; S=270.8yd2.

p) q) r) s)
8cm
4cm
3cm 3cm
1.5cm
3cm
Sol: V=376.3in3; S=401.3in3 V=1357cm3; S=678.6cm2 V=12cm3; S=34.6cm2 V=75.4cm3; S=108.8cm2.

t) u) v) w)
x

Sol: V=179.1yd3, S=169,4yd2 V=75cm3, S=105cm2 V=140ft3, S=226ft2 V=256’24in3, S=1017.9in2.

92
5. Count the number of vertices, edges and faces. Check it using Euler’s formula.
a) Dodecahedron b) Icosahedron

c) Elongated TriangularPyramid d) Cuboctahedron

e) Truncated Cube f) Square Cupola

g) Great Rhombicuboctahedron h) Pentagonal Rotunda

i) Rhombicosidodecahedron j) Octagonal Antiprism

k) Augmented pentagonal prism l) The hole causes Euler’s formula be equal to 0!

Solutions:
a) F = 12, E = 30, V = 20. b) F = 20, E = 30, V = 12. c) F = 7, E = 12, V = 7.
d) F = 14, E = 24, V = 12. e) F = 14, E = 36, V = 24. f) F = 10, E = 20, V = 12.
g) F = 26, E = 48, V = 24. h) F = 17, E = 35, V = 20. i) F = 62, E = 120, V = 60.
j) F = 18, E = 32, V = 16. k) F = 10, E = 19, V = 11. l) F = 16, E = 32, V = 16.

93
GEOMETRIC BODIES AND VOLUMES - WORD PROBLEMS

1. Nara wants to determine how much ice it will take to fill her cooler. If the cooler has a
length of 22 inches, a width of 12 inches, and a height of 10 inches, how much ice will her
cooler hold? [*] 1 inch=2.54cm

2. The cargo-carrying part of Billy’s truck has a length of 8.3 meters,


a width of 3 meters, and a height of 4.2 meters. What is the
maximum volume of sand that Billy’s truck can carry?

3. Alexia’s bathroom has a tub (bañera) in the shape of a


rectangular prism with a length of 1.5 metres, a width of 0.5 metre, and a height of 0.4
3
metre. How many cubic meters of water can it hold? How many litres? [*] 1m =1000l.

4. A box of tissues (pañuelo) has a length of 11.2 centimetres, a width of


11.2 centimetres, and a height of 13 centimetres. What is its volume?

5. A cereal box has a length of 8 cm, a width of 1 cm, and a height of 12 cm. What is the
volume of the cereal box? [Sol: 169.75cm3]

6. A cylindrical water tank has a diameter of 5.3 meters and a height of 9 meters. What is the
maximum volume that the water tank can hold? [Sol: 198.5565m3=198,556.5 litres]

7. A can of corn has a diameter of 6.6 centimetres and a height of 9.9


centimetres. How much corn can the can hold? [Sol: 338.7cm3]

8. Tamara wants to determine the maximum capacity of a cylindrical bucket


(cubo) that has a radius of 12cm and a height of 30cm. What is the capacity
of Tamara’s bucket? [Sol: 13,571.7cm3=13.5717 litres]

9. Rodolfo is designing a new, cylindrical drinking glass. If the glass has a diameter of 8
centimeters and a height of 12.8 centimeters, what is its volume? [Sol: V=643.4cm3]

10. A can of paint is 15 centimeters high and has a diameter of 13.6 cm. What is the volume of
the can? [Sol: V=2,179cm3]

11. Jane needs to buy insulation (pintura aislante) for the inside of a truck container. The
container is a rectangular prism 15 feet long, 8 feet wide, and 7 feet high. How much
insulation should Jane buy if all inside surfaces except the floor are to be insulated?

12. A packaging company needs to know how much cardboard will be required to make boxes
18 inches long, 12 inches wide, and 10 inches high. How much cardboard will be needed
for each box if there is no overlap (solapar) in the construction? [Sol: 1032in2]

94
13. What is the area of the label (etiqueta) on a box of oatmeal (harina de avena) with a radius
of 9.3 centimetres and a height of 16.5 centimetres? Round to the nearest tenth.

14. Betty wants to know the total surface area of the tread (dibujo del
neumático) on one of her tires. If the diameter of the tire is 18 inches and the
width of the tire is 5 inches, what is the total surface area of the tire’s tread?

15. How much sheet metal is required to make a cylindrical trash can with a
diameter of 2 feet and height of 4 feet? How much if we include the top?

16. How much steel is needed to make a hollow pipe (tubería hueca) with a radius of 3 inches
and a height of 15 inches? Round to the nearest tenth. [Sol: 282.7in2]

17. If a cylindrical water bottle has a radius of 4 centimetres and length of 13 centimetres,
3
how many cubic centimetres of water will it hold? [Sol: V = 653.4513cm ]

18. Hannah keeps her favorite toys in the closet (armario) of her bedroom. If the closet is 2
meters high, 2 meters long, and 3 meters wide, how many cubic meters of toys can the
3
closet hold? [Sol: 12m ]

19. Using her new telescope, Lauren discovers a spherical meteor passing between the Earth
and the Moon and measures its radius as 4 kilometers. How many cubic kilometers are in
3
the meteor? [Sol: V = 268.0826km ]

20. Juan piled his dirty clothes 3 meters high in a laundry basket that was 2m long and 1m
3
wide. How many cubic meters of dirty laundry did he have to wash? [Sol: 6m ]

21. Logan measured a cubic storage room to be 4 meters wide. How many square meters
3
does he have for storage space? [Sol: 64m ]

95
12. PROBABILIDAD
 Experimento: suceso que podemos repetir bajo condiciones semejantes. Puede ser:
Deterministas: sólo hay un resultado posible.
Aleatorio: si hay varios resultados posibles. Cada uno de estos resultados se llama
suceso elemental. El conjunto de todos los resultados posibles se llama Espacio Muestral (E).
La unión de varios sucesos elementales se llama suceso compuesto.
 Experimento compuesto: cuando el experimento puede dividirse en otros más
sencillos. Para calcular el espacio muestral podemos usar un diagrama de árbol.
Ejemplo: “lanzar dos monedas” puede descomponerse en: 1ª moneda y 2ª moneda.
- Si no necesitamos todo el espacio muestral, podemos podar el árbol: no dibujar las ramas
que no nos interesan.

 Probabilidad: es el grado de creencia que tenemos de que ocurra un suceso. La medimos en


porcentaje, fracción o número (entre 0 y 1).
Un suceso de probabilidad 100%, se llama suceso seguro, y el de probabilidad 0% se llama
suceso imposible.

 Cálculo de probabilidades:

- Regla de LaPlace: , pero sólo puede usarse cuando podemos afirmar


que todos los casos tienen la misma probabilidad (son equiprobables).

- Diagramas de árbol: la probabilidad de cada suceso se calcula multiplicando la de cada rama.

- Ley de los grandes números: la probabilidad es la frecuencia relativa, siempre que se haya
repetido el experimento muchas veces.

97
PROBABILITY
 Experiment: test we can repeat under similar conditions. It can be:
Deterministic: there is only one possible result.
Random: There is more than one possible result. Each of them is called simple event.
The set of all possible outcomes is called the Sample Space (S).
If an event has more than one elements then it is called a compound event.
 Compound Experiment: when the experiment can be break into some more
simple ones. We can use a tree diagram to calculate the sample space.
Example: “tossing two coins” may be broke into: 1st coin and 2nd coin.
- If we do not need the whole sample space we can prune the tree: do not draw the branches
we are not interested in.

 Probability: degree of likelihood that something will happen. We can measure it in


percentage, fraction or number (between 0 and 1).
A 100% probable event is called sure event, and a 0% probable event is called impossible
event.

 Computing probabilities:

- LaPlace’s rule (rule of Succession): , it can only be used when we


can state that all outcomes are equally likely (are equiprobable).

- Tree diagrams: the probability of each event is computed multiplying the probability of each
branch.

- Law of Large Numbers: the relative frequency is the probability, as long as we repeat the
experiment many times.

98
Probability - Word Problems
1. There are 8 children in Christopher's preschool class. During free time yesterday, 2 of
them chose to paint. What is the probability that a randomly selected child chose to paint
yesterday?

2. On the last day of a Shakespeare class, an English teacher asked her students which play
they liked most. Out of the 14 students, 4 liked Macbeth best. What is the probability that
a randomly selected Shakespeare student likes Macbeth best?

3. Kylie's father has a large collection of ties. He has 18 ties, and 9 of them are red. If Kylie's
father chose a tie at random, what is the probability that the tie would be red?

4. You spin the spinner once. What is P(6)? What is P(not even)?

5. You roll a 6-sided die. What is P(3 or greater than 4)?

6. A grocery store recently sold 12 cartons of yogurt, of which 6 were blueberry. What is the
experimental probability that the next carton of yogurt sold will be blueberry?

7. Chris is birdwatching at the coast. He has seen 4 gulls (gaviota) out of 10 total birds. What
is the experimental probability that the next bird Chris sees will be a gull?

8. Wyatt surveyed (hacer una encuesta) 13 students at his school and found that 2 of them
planned to take art as their next elective (optativa). What is the experimental probability
that a randomly selected student plans to take art?

9. Restaurants often slip (deslizar) takeout (comida para llevar) menus under Bob's
apartment door. So far (hasta ahora), Bob has collected 15 menus, including 5 for Indian
food. What is the experimental probability that the next menu slipped under Bob's door
will be from an Indian restaurant?

10. If you roll a 6-sided die (dado) 6 times, what is the best prediction possible for the
number of times you will roll a three? What if your roll it 12 times?

11. If you spin the spinner 15 times, what is the best prediction possible for the
number of times it will land on a light (claro) place? And spinning 12 times?

12. Murphy surveyed some students at his school about their favourite colours: Brown (2),
Orange (3), Green (4), Gray (3) [AmE: gris]. If Murphy surveys 16 more students, how many
of them should he expect to pick gray, based on past data?

13. Bethany pulled a number of coloured marbles (canica) from a large bag. Blue (4), White
(1), Brown (5). Based on experimental probability, how many of the next 16 marbles
selected should you expect to be brown marbles? [Sol: 8 m. Hint: Compute P(Brown)=1/2.]

99
14. There are 50 socks in Patricia’s drawer. There are 25 black socks, 10 blue socks, 10 orange
socks and 5 striped (a rayas) socks.
a) Patricia picks a sock without looking. Write the sample space and find the probability of
each simple event. [Sol: S={Bk,Bl,Or,St}, P(Bk)=50%, P(Bl)=20%, P(Or)=20%, P(St)=10%]

b) She picks two socks without looking. What is the probability of picking the same colour?
[Hint: tackle it using a pruned tree]. [P(Same colour)=24.5%+3.7%+3.7%+0.82%=32.72%].

And the probability of picking different colours? [P(Different colours)=100-32.72=67.28%].

15. Find how many outcomes are possible, without drawing a tree:

a) You roll a die and flip a coin. [Sol: 12]

b) You roll a die, pick a marble and pick a card. [Sol: 24]

c) You flip a coin, spin the spinner, and pick a card. [Sol: 30]

d) You spin each spinner and pick a marble. [Sol: 24]

16. The table below lists the number of students at secondary by grade and gender.
- One student is chosen at random. Determine the probability of: Boys Girls Totals
1ºESO 57 51 108
a) P(Girl)= 52% b) P(Boy)= 48% c) P(1ºESO)= 36% 2ºESO 42 57 99
3ºESO 45 48 93
d) P(1ºESO, Boy)= 19% e) P(3ºESO,girl)= 16% Totals 144 156 300
- We’ve got some information about the pupil chosen. Find the probabilities now:
f) Knowing that a boy has been chosen, find the probability that he is in 3ºESO. [P=31.25%].
g) Knowing that someone from 2ºESO has been chosen, probability that it is a boy. [P=42.42%].
h) Knowing that a girl has been chosen, find the probability that she is in 2ºESO. [P=36.54%].

17. You roll a die, pick a card, and find the sum.
a) How many different sums are possible? [Sol: 7 possible sums; between 5 and 11]
b) What is the probability of obtaining 6? [Sol: P=16.7%]

18. You spin each spinner and find the sum.


a) How many different sums are possible? [Sol: 6 outcomes; from 11 to 16]
b) What is the probability of obtaining 12? [Sol: P=20%]
c) What is the probability of a sum greater or equal to 15? [Sol: P=30%]

100
2º ESO Mathematics Trial Exam IES Extremadura
Bilingual section
Surname _________________________________________________ Mark
Name __________________________________

VOLUMES
1. (3 pts.) Find the volume of the figure. Write each solid’s name.
2. (3 pts.) Use the net to find its surface. 8cm
2cm
2cm 2.24cm
9cm
4cm 4cm
3cm x

2.24cm 4cm 4cm


4cm
x
2cm
4cm
9cm

PROBABILITY
3. (1 pt.) Isabelle runs a day-care centre (guardería infantil). So far this year, the enrolment
(inscripción) has consisted of 8 babies and 4 children of other ages. Considering this data:
a) What is the experimental probability that the next children to enrol consist of a baby?

b) How many of the next 21 children to enrol should you expect to be babies?

4. (3 pts.) Clara’s got a bag containing 5 red marbles, 4 green marbles and 3 yellow marbles.

a) Write the sample space for the experiment “draw two marbles from the bag”, and take
note of their colour.
b) Answer, without drawing the tree: If we pick three marbles, how many outcomes are
possible? (Suppose the order is important).

c) If we pick three marbles (without replacement), find the probability that “all three are
of the same colour”.

101

Potrebbero piacerti anche